Search This Blog

Thursday, January 14, 2010

MCCQE1

ANESTHESIA


1. Which of the following is the proper positioning for

intubation?

a) extension of the lower C-spine and extension of the

atlanto-occipital joint

b) flexion of the lower C-spine and extension of the

atlanto-occipital joint

c) flexion of the lower C-spine and flexion of the

atlanto-occipital joint

d) open the mouth only half way to get a good look at

the cords

e) have the patient’s head on a high pillow to facilitate

intubation

2. Which of the following is NOT part of rapid sequence

induction?

a) patient breathes 100% O2 for 3-5 minutes prior to

induction

b) the patient receives bag ventilation immediately

following induction

c) the Sellick manoeuvre

d) pressure placed on the cricroid cartilage

e) muscle relaxant given

3. Which of the following methods is sufficient to

determine if the EET has passed through the cords?

a) chest movement

b) absence of abdominal distension

c) condensation of water vapour in the EET tube

during expiration

d) CO2 in exhaled gas as measured by capnogragh

e) refilling of reservoir during exhalation

4. Which of the following is CORRECT?

a) extracellular fluid volume (ECF) equals 2/3 of total

body water

b) ECF volume deficit will cause hypotension and

bradycardia

c) ECF volume expansion will cause increased JVP

and S3

d) moderate dehydration means 10% of the ECF

volume is lost

e) [Na+] determines ECF volume

5. Which of the following is NOT a transfusion reaction?

a) fever

b) hypercalcemia

c) anaphylaxis

d) intravascular hemolysis

e) extravascular hemolysis

6. Which of the following is NOT a side-effect of opioid

analgesics?

a) diarrhea

b) constipation

c) nausea

d) biliary spasm

e) respiratory depression

7. Opioid analgesics:

a) Prevent the release of inflammatory/pain mediators

at the site of injury

b) Block synaptic transmission along the pain pathway

c) Stimulate respiration and increase blood pressure

d) Are rarely associated with tolerance

e) Cause papillary dilation

8. The most potent inhalation induction agent that is

commonly used is:

a) enfluane

b) isoflurane

c) sevoflurane

d) nitrous oxide (N2O)

e) halothane

9. Contraindications to thiopental include all following

EXCEPT:

a) cardiac failure

b) porphyria

c) bowel obstruction

d) potential difficult intubation

e) status asthmaticus

10. The following is a depolarizing muscle relaxant?

a) Rocuronium

b) Mivacurium

c) Succinylcholine

d) Tubocurarine

e) Pancuronium

11. Which of the following is FALSE? Epidural analgesia:

a) causes uterine relaxation

b) controls blood pressure in preeclampsia

c) can cause urinary retention

d) can contribute to the effect of caval compression

e) can cause itching

12. All but one of the following increases the likelihood of

regurgitation at the time of induction:

a) obesity

b) anxiety

c) upper airway obstruction

d) increased esophageal sphincter pressure

e) hiccups

13. Which of the following is used in the treatment of

malignant hypertension:

a) succinylcholine

b) ketamine

c) dantrolene

d) tubocurarinee

e) neostigmine

14. Regarding pre-operative investigations, all of the following

are true EXCEPT:

a) EKG recommended for those > 40 year old

b) pulmonary function test (PFT’s) for those with COPD

c) Hb and urinanalysis are required for every surgical

patient as indicated in the Public Health Act

d) hospital policy dictate the indications for tests such as

CXR

MCCQE 2002 Review Notes Sample Questions – 3

CARDIAC AND VASCULAR SURGERY

1. Intermittent Claudication is:

a) a reproducible discomfort in exercising muscle

groups

b) secondary to occlusive arterial disease

c) a relatively benign condition when treated with risk

factor modification and exercise

d) may be life-style limiting

e) all of the above

2. Ischemic rest pain is:

a) unremitting pain in the most distal portion of the

affected extremity

b) a precursor to gangrene and limb-loss

c) associated with pallor on elevation and rubor on

dependency

d) should be treated with aggressive revascularization

surgery

e) all of the above

3. An arterial ulcer can be described as:

a) red granulating base. Located over a weight bearing

area. Non-painful

b) red granulating base. Heaped up border with

venous engorgement. Located in the gaiter

distribution

c) whitish, necrotic base. “Punched out” appearance.

No evidence of healing, painful. Located usually on

the distal parts of the extremity

d) all of the above

e) none of the above

4. An aneurysm can be defined as:

a) an artery greater than 5 cm in diameter

b) a localized enlargement of a vessel greater than

1.5X its expected diameter

c) an artery greater than 3 cm in diameter

d) a blood vessel greater than 1.5X its expected

diameter

e) an out-pouching of the aorta

5. The clinical findings of a ruptured abdominal aortic

aneurysm include:

a) pulsatile abdominal mass

b) hypotension

c) back pain

d) hematemesis

e) A, B, and C

f) all of the above

4 – Sample Questions MCCQE 2002 Review Notes

CARDIOLOGY

1. All of the following regarding the management of

heart failure are true EXCEPT:

a) digitalis is indicated in atrial fibrillation

resulting in heart failure

b) the use of ß-blockers in acute CHF has been

shown to reduce mortality

c) calcium channel blockers add no proven survival

benefit in patients with CHF

d) oral inotropes have a detrimental effect on survival.

e) ACE inhibitors improve survival in

asymptomatic patients with LVEF < 35%

2. Regarding pericardial disease, which of the following is

INCORRECT?

a) the ECG changes of acute pericarditis include PR

segment depression and ST segment elevation

b) autoimmune pericarditis is not associated with

acute MI

c) a prominent “x” descent might be expected in the

JVP waveform of a patient with a pericardial effusion

d) tamponade may result in hypotension, elevated

central venous pressure, and pulsus paradoxus

e) a relatively small pericardial effusion may result in

tamponade if it develops rapidly

3. Regarding syncope:

a) the etiology of as many as 50% of cases is never

identified

b) vasovagal syncope (the common faint) results from a

combination of peripheral vasodilatation and relative

bradycardia

c) a common cause of syncope is a cortical stroke

d) the final common pathway of most causes of

syncope is usually generalized cerebral

hypoperfusion

e) cough, micturition, and defecation syncope all result

in part from impaired venous return to the heart

4. Which of the following statements is INCORRECT?

a) chronic sympathetic nervous system activation

causes long-term changes in the myocardium in

heart failure

b) the development of heart failure requires an

initiating myocardial insult

c) dilated cardiomyopathy is a common cause of CHF

d) poor ventricular compliance results in an S3

e) Cheyne-Stokes breathing is frequently seen in

patients with CHF

5. Which of the following statements about valvular cardiac

disease is CORRECT:

a) pulmonary valve disease is a leading cause of

morbidity and mortality

b) rheumatic heart diseases is a minor cause of mitral

stenosis

c) mitral regurgitation leads to LV dilatation

d) radiation of a systolic murmur to the clavicle is not

commonly seen in aortic stenosis

e) mitral valve prolapse leads to a lower JVP

6. Which of the following provides symptomatic relief but

has not ben shown to increase longevity in the context of

CHF:

a) ACE inhibitors

b) loop diuretics

c) beta-blockers

d) spironolactone

e) hydralazine and nitrates

7. The least serious contraindication to Thrombolytic

Therapy in acute myocardial infarction is:

a) cerebral hemorrhage

b) active bleeding

c) acute pericarditis

d) chronic liver disease

e) aortic dissection

8. Which of these is NOT a peripheral sign of infective

endocarditis:

a) Osler's nodes

b) splinter hemorrhages

c) Janeway lesions

d) clubbing

e) palmar erythema

9. Which of the following is not one of the major Jone’s

criteria:

a) pancarditis

b) subcutaneous nodules

c) polyarthritis

d) Huntington’s chorea

e) erythema margination

10. Which of the following is TRUE regarding the JVP:

a) 3rd degree heart block is associated with cannon

a waves

b) a positive Kussmaul sign is not correlated with

constrictive pericarditis

c) C-V waves are seen in tricuspid stenosis

d) atrial fibrillatoin does not affect the JVP waveform

e) the “y” decent occurs during ventricular systole

MCCQE 2002 Review Notes Sample Questions – 5

COMMUNITY HEALTH

1. One rationale for universal hepatitis B vaccination in

infants is that:

a) the vaccine is not effective in preventing infection in

high-risk adults

b) no risk factors can be identified in 25% of hepatitis B

cases

c) hepatitis B infections occur commonly in children

d) the HBV vaccine is contraindicated in at-risk

pregnant women

e) most chronic carriers contract hepatitis B during early

childhood

2. Polyvalent pneumococcal vaccine is NOT

RECOMMENDED in patients:

a) with sickle cell anemia

b) with HIV infection

c) who take prophylactic penicillin

d) under 2 years of age

e) who are pregnant or considering pregnancy in the

next three months

3. Which one of the following is caused by chronic tobacco

smoking?

a) increased throat pain during streptococcal infection

b) increased ciliary action

c) increased anti-protease activity

d) hyperplasia of respiratory epithelium

e) increased risk of ulcerative colitis and endometrial

carcinoma

4. You are director of occupational health for a corporation

that has many employees aged over 45 who smoke one

or more packs of cigarettes daily and are at increased

risk for lung cancer. What strategy for the early detection

of lung cancer in asymptomatic individuals would you

recommend?

a) no strategy has been shown to be effective in

reducing mortality

b) chest x-ray and sputum cytology every 6 months for

high-risk employees

c) annual chest x-ray and sputum cytology for high-risk

employees

d) annual chest x-rayand sputum cytology for all

employees

e) annual chest x-ray for all employees

5. Regarding obesity:

a) exercise, diet and behavioural therapy result in

sustained weight loss in 50% of patients

b) obesity is the leading cause of preventable death in

industrialized countries

c) the periodic health exam recommends routine diet

counselling and measuring BMI for all individuals

d) even a small weight loss is associated with benefit

e) switching from soft drinks sweetened with sugar to

those with artificial sweetners results in fewer

calories consumed and sustained weight loss for

most patients

6. Regarding cocaine abuse:

a) cocaine abuse is primarily confined to lower social

classes, and is often associated with opiate abuse

b) cocaine is well absorbed via the oral route

c) physical dependence is common, and physical

withdrawal symptoms are severe

d) cocaine may cause toxic psychosis and hypertension

e) cocaine has no current medical indications

7. Pneumonia:

a) is the third most common infective cause of death in

the developed world

b) is very rarely caused by Gram-negative organisms in

elderly patients

c) is treated at home in less than 50% of patients

d) pleuritic pain is a poor prognostic sign

e) a poor white cell response is an ominous sign

8. A case control study is designed to analyse a suspected

hypothetical association between the development of

mania and migraine treatment. This study:

a) can derive and estimate of the risk of an individual

developing the mania as a consequence of migraine

treatment

b) will demonstrate that any association found is likely

to be causal

c) will need controls who are chosen at random from the

general population

d) will necessitate careful follow-up of a group of patient

receiving migraine treatment, and a not treated

control group

e) will not give biased results if all the patients have

been carefully assessed by a senior clinician

9. A double blind trial is planned to compare the utility of

glyburide and metformin in the treatment of diabetes

mellitus. The main reasons for randomising patients are:

a) so that the number of subjects in each group will be

identical

b) so that the two patient groups will have similar

prognostic features

c) so that the statistian will not analyse the data in a

biased fashion

d) so that the investigator does no know in advance

what therapy which patient will receive

e) to prevent the clinician knowing which drug the

patient is taking

10. Which statement is TRUE?

a) the assignment of a gender is a nominal variable

b) histogram is a useful way of showing how a particular

variable is changing in respect to time

c) the mean of a sample of positive numbers is

invariably greater than the median value

d) the mean of a large sample will always increase in

size as the size of the sample increase

e) approximately 70% of data points are both within one

standard deviation of the mean and larger than the

mean of a normally distributed data set

6 – Sample Questions MCCQE 2002 Review Notes

COMMUNITY HEALTH . . . CONT.

11. Which clinical picture most likely corresponds to a

patient who has ingested “two bottles of aspirin”?

a) arterial thrombosis and hypothermia

b) hyperpnea, tinnitus, and respiratory alkalosis

c) metabolic acidosis, hypokalemia, hyperglycemia

d) weakness, ataxia, drowsiness

e) facial pallor followed later by hepatic tenderness

12. Who should be screened for lead poisoning in Canada?

a) all children with developmental delay

b) all children living in housing built before 1960

c) all children living in housing in high-risk districts

d) children with unexplained symptom complex of

abdominal pain, anorexia, anemia, ataxia, and

slurred speech

e) none of the above

13. Each of the following patients has a 5-mm reaction to a

5TU PPD skin test. Who should receive therapy for

tuberculosis?

a) an intravenous drug user known to be

HIV-seronegative

b) a Vietnamese refugee

c) a 30-year-old long-term resident of a mental

institution

d) a 40-year-old with diabetes mellitus

e) a 40-year-old female with known HIV infection

14. A 54-year-old white female who is new to your practice

presents with a viral upper respiratory infection. She

insists that three days of penicillin is the only thing that

ever cures it. You explain your reasons for not using an

antibiotic, but she continues to insist. You should:

a) treat her symptomatically but do not give antibiotics

b) refer her to a physician in your community who has

a reputation for frequently using antibiotics

c) have the patient sign a medical release before

giving her a script for gentamycin

d) tell her that if she is not better in three days you will

give her an antibiotic as a compromise

15. Fetal infection with Hepatitis B is most likely during the:

a) first trimester

b) second trimester

c) third trimester

d) throughout pregnancy

e) Hepatitis B is not vertically transmitted

16. Appropriate treatment of a neonate born to a Hepatitis

B positive mother is:

a) hepatitis b immune globulin (HBIG) at birth

b) HBIG at birth and Hepatitis B vaccine at 2, 4, and 6

months of age

c) HBIG and Hepatitis B vaccine at birth, 1, 6 months of

age

d) HBIG at birth and Hepatitis B vaccine at birth, 1 and

6 months of age

e) hepatitis B vaccine at 2, 4, and 6 months of age

17. With regards to the vertical transmission of Hepatitis B:

a) there is a less than 1% risk if the mother is

asymptomatic and HBsAG+

b) can be transmitted through the placenta and

through breast milk only

c) risk of vertical transmission is greater than 80% if

mother is HBsAg+ and HBcAg+

d) chronic active hepatitis B has no effect on prematurity

e) hepatitis B vaccine should not be given to a pregnant

woman

18. Vertical transmission of HIV can be reduced by giving AZT:

a) during pregnancy

b) during labour

c) at the delivery

d) to the neonate

e) all of the above

MCCQE 2002 Review Notes Sample Questions – 7

DERMATOLOGY

1. All of the following have been implicated in the

pathogenesis of acne vulgaris EXCEPT:

a) androgen stimulated production of sebum

b) Propionibacterium acnes

c) consumption of sugars, fats and oils

d) hyperkeratinization of follicle lining

2. Which of the following medications is not indicated for

the treatment of acne vulgaris?

a) benzoyl peroxide

b) topical erythromycin

c) adapalene

d) tazarotene

e) metronidazole

3. Which of the following is not a feature of perioral

dermatitis?

a) papules and inflammatory plaques

b) rim of sparing round vermillion border of lips

c) predominantly males

d) teenagers to mid-adulthood

a) treated with topical metronidazole

4. You have been referred a 2 month old with atopic

dermatitis. Where are the most likely sites of the

dermatitic eruptions in an infant of this age?

a) peripheral fingers and toes

b) flexural creases of elbows and wrists

c) face and extensor surfaces of limbs

d) diaper distribution

5. You have been referred an 80 year old man with a

history of venous insufficiency, who complains of a

constant irritation of his lower legs of 6 months

duration. The most likely diagnosis is:

a) seborrheic dermatitis

b) stasis dermatitis

c) atopic dermatitis

d) allergic contact dermatitis

6. The cause of “cradle cap” in infants is most commonly:

a) atopic dermatitis

b) seborrheic dermatitis

c) pityriasis rosea

d) alopecia areata

7. The most common causative agents of Impetigo

Vulgaris are:

a) pseudomonas and streptococcus

b) fungi

c) GABHS and Staphylococcus aureus

d) GABHS and Group B Streptococcus

e) Mycoplasmae

8. Staph aureus group II exfoliating toxin is associated with

a) bullous impetigo in axillae and groin folds

b) psoriatic knee patches

c) facial atopic dermatitis

d) vitiligo

9. Which of the following is NOT a true difference between

erysipelas and cellulitis?

Erisypelas Cellulitis

a) Upper dermis Lower dermis

and subcutaneous fat

b) Well demarcated Poorly demarcated

c) Group A Strep as Dermatophyte

causative agent as causative agent

d) commonly on face commonly on extremities

10. Primary syphilis is described by the following EXCEPT:

a) excruciatingly painful buttonlike papule

b) regional non-tender lymphadenopathy

c) VDRL negative initially

d) Treated with 2.4 million units benzathine penicillin

G given intramuscularly

11. Hand-Foot-and-Mouth Disease is:

a) common in the elderly

b) caused by staphylococcus aureus

c) caused by Coxsackie A16 virus

d) chronic

e) treated with antibiotic medications

12. Which of the following is not associated with a

dermatophytic infection?

a) Tinea capitis

a) Tinea Corporis

a) Condylomata acuminata

a) Onychomycosis

13. Which of the following typically presents after acute

streptococcal pharyngitis?

a) atopic dermatitis

b) guttate psoriasis

c) pityriasis rosea

d) plaque psoriasis

14. Which of the following is associated with gluten

intolerance?

a) Bullous pemphigoid

b) Pemphigus vulgaris

c) Dermatitis herpetiformis

d) Porphyria cutanea tarda

15. Which is not a feature of lichen planus?

a) purple

b) polygonal

c) pruritic

d) purulent discharge

e) peripheral distribution

16. All of the following are characteristics of basal cell

carcinoma EXCEPT:

a) associated with UV radiation

b) pearly nodule with telangiectasia

c) located on body surfaces not exposed to sunlight

d) spread by local invasion

e) age of onset >4 0

8 – Sample Questions MCCQE 2002 Review Notes

DERMATOLOGY . . . CONT.

17. Which is NOT a feature of a cutaneous drug eruption?

a) toxic epidermal necrolysis

b) angioedema

c) phototoxicity

d) hypopigmentation

e) anaphylaxis

18. Which is NOT a feature of vitiligo and its management?

a) associated with thryoid disease and diabetes

mellitus

b) more common in blacks than whites

c) treatment options include PUVA therapy

d) destruction of melanocytes

e) well-demarcated white macules

19. All of the following are skin diseases associated with

diabetes mellitus EXCEPT:

a) eruptive xanthomas

b) tinea pedis

c) acanthosis nigricans

d) necrobiosis lipoidica

e) pruritis

20. Malignant melanoma involving the papillary dermis is

Clark Level:

a) I

b) II

c) III

d) IV

e) V

21. All of the following are treatments for non-scarring

alopecia EXCEPT:

a) spironolactone

b) minoxidil

c) hair transplantation

d) intralesional triamcinalone

e) finasteride

MCCQE 2002 Review Notes Sample Questions – 9

EMERGENCY MEDICINE

1. A patient presents with a puncture wound to the hand

that occurred 12 hours ago. He is not certain if he has

been vaccinated for tetanus. Which of the following is the

proper management of this patient with respect to

tetanus prophylaxis?

a) tetanus toxoid

b) tetanus immune globulin

c) both tetanus toixoid and tetanus immune globulin

d) no prohylaxis required

2. A 34 year-old otherwise healthy woman presents in the

trauma room after being involved in a motor vehicle

collision. She is strongly suspected to have internal

bleeding. Her heart rate is 130, blood pressure 85/50,

and respiratory rate is 40. Which of the following are

likely TRUE?

a) she has lost < 1,000 cc of blood

b) she will have normal urine output

c) fluid replacement should consist of crystalloid and

blood

d) she has lost > 40% of her blood volume

3. A patient presents to the emergency department. On

examination he requires a sternal rub to open his eyes.

He is muttering incomprehensively and withdraws his

hand when a painful stimulus is applied. His Glascow

Coma Scale score is:

a) 7

b) 8

c) 9

d) 10

4. With respect to question 3, the most appropriate

management of this patient’s airway is:

a) none

b) oropharyngeal airway

c) nasopharyngeal airway

d) endotracheal intubation

e) cricothyroidotomy

5. The anticholinergic and sympathomimetic toxidromes

shareall of the following features EXCEPT:

a) hyperthermia

b) maydriasis

c) tachycardia

d) diaphoresis

6. Which of the following is NOT true regarding the

management of asthma patients in the emergency room:

a) patients unable to speak with an O2 sat < 90%

should be intubated

b) peak flow meters are the best way to assess

response to therapy

c) patient education about the proper use of puffers

is an important aspect of treatment

d) anti-cholinegics are the first line agents in the

treatment of mild asthma

e) a silent chest on auscultaton indicates an

emergency and requires immediate treatment

7. Which of the following is TRUE regarding hypertensive

emergencies?

a) there is evidence of end organ damage

b) renal failure can be both a cause and an effect of

hypertensive emergencies

c) the goal of treatment is to lower the blood pressure

to normal within 30-60 minutes

d) fundoscopic findings may include papilledema and

hemorrhages

e) all of the above

8. In a head trauma patient the most important feature on

history is:

a) seizure

b) loss of consciousness

c) headache

d) nausea and vomiting

9. According the Ottawa Ankle Rules, an ankle radiographic

series is required in all of the following situation EXCEPT:

a) bony tenderness to palpation along the posterior

edge of the lateral maleolus

b) bony tenderness to palpation along the posterior

edge of the medial malleolus

c) bruising over either malleolus

d) inability to weight bear both immediately

following the injury and in the emergency

department

10. Which of the following is NOT true with respect to

hypothermia?

a) chest compressions should be deferred up to

1 minute until it is certain that the patient is

pulseless as they can precipitate ventricular

tachycardia

b) pupils become fixed and dilated at core body

temperature below 25ºC

c) ventricular fibrillation becomes a risk at core

body temperatures < 30ºC

d) the patient should be rewarmed slowly to

avoid an afterdrop

11. Which of the following is FALSE regarding adult patients

presenting to the emergency department following a

sexual assault?

a) they will often present with chief complaints other

than sexual assault

b) physicians are legally obligated to report the

assault to the police even if the victim does not

wish it reported

c) the patient should have folllow up with an MD at

a rape crisis centre within 24 hours

d) pregnancy and sexually transmitted disease

prophylaxis should be offered if appropriate

10 – Sample Questions MCCQE 2002 Review Notes

EMERGENCY MEDICINE . . . CONT.

12. Which of the following is NOT true regarding allergy and

anaphylaxis?

a) they are IgE mediated immune response to

antigens

b) epinephrine is the first line agent for the treatment

of minor skin eruptions to topical agents

c) signs and symptoms of an allergic reaction vary

from minor cutaneous eruptions to cardiovascular

collapse, bronchospasm and laryngeal edema

d) the ABC’s are the first step in the treatment of

any allergic reaction

13. Which of the following is FALSE with respect to chest

pain?

a) all patients presenting with chest pain should

receive an ECG

b) a normal ECG and normal cardiac enzymes rule

an ischemic event

c) a careful history is essential in determining the

etiology of the chest pain

d) reproducible chest pain on palpation of the chest

wall does not rule out an acute MI

14. Which of the following toxins is NOT a cause of an

increased anion gap metabolic acidosis?

a) methanol

b) ethylene glycol

c) salicylates

d) iron

d) benzodiazepines

15. Which of the following toxins is NOT matched to its

specific treatment?

a) acetaminophen - N-acetylcysteine

b) carbon monoxide - hyperbaric oxygen

c) heroin - naloxone

d) diazepam - bicarbonate

e) methanol - ethanol

MCCQE 2002 Review Notes Sample Questions – 11

ENDOCRINOLOGY

1. A patient complains of a non-tender mass over the

thyroid region on the left side of her neck. Concerned

about a thyroid disorder, you order the appropriate

investigations. The results are as follows:

TSH: 6.0

Free T4: 20.2

Thyroid antibodies: none

RAIU: No “hot” spots seen

The next investigation(s) you choose to do are:

a) watch and wait for 3-6 months

b) FNA

c) surgical biopsy

d) trial of L-thyroxine therapy for 6 months

e) none of the above

2. An 8 year-old boy is brought to the office because his

mother is concerned he is entering puberty already.

You examine him and note the beginnings of facial hair,

axillary hair and Tanner stage 2 external genitalia.

Choose the set of investigations you initially want to do:

a) CBC, lytes, testosterone, bone age, CT head

b) FSH, LH, testosterone, lytes, bone age, DHEA-S

c) FSH, LH, testosterone, cortisol, DHEA-S,

11-OH progesterone, bone age

d) lytes, testosterone, DHEA-S, 17-OH progesterone,

cortisol, bone age

3. A 2 month-old boy has a Ca of 1.80 (corrected) after an

assessment for FTT. Mother informs you she has been

consistently breastfeeding without trouble as

corroborated by a visiting nurse. What is at the top of

your differential?

a) malabsorption

b) pseudohypoparathyroidism

c) mother didn’t supplement with DiVisol

(Vit D supplement)

d) DiGeorge syndrome

4. The “triple bolus” test of pituitary function works by a

rapid succession of IV constituents as follows:

a) insulin – hypoglycemia mediated rise in

GH and ACTH

LHRH – rise in LH and FSH

TRH – rise in TSH and PRL

b) CRH – rise in GH and ACTH

LHRH – rise in LH and FSH

TRH – rise in TSH and PRL

c) estrogen – rise in LH, drop in FSH and PRL

insulin – rise in GH and ACTH

TRH – rise in TSH

d) cosyntropin – rise in GH and ACTH

GHRH – rise in LH and FSH

TRH – rise in TSH and PRL

5. A 58 year-old man with a past history of a

parathyroidectomy for primary hyperparathyroidism is

now in your office complaining of headaches worse in the

AM (made worse by a small MVA he credits to a loss of

peripheral vision). You plan to:

a) send to the Emergency Department for an

immediate CT head

b) check his calcium to ensure there’s no remaining

parathyroid tissue

c) check for a pheochromocytoma (which you know

causes headaches) because you are concerned he

has MEN I syndrome

d) check for a homonymous hemianopia because you

are worried about a pituitary tumour

e) check for a bitemporal hemianopia because you are

worried about a pituitary tumour

6. Which of the following is not associated with thyroid

disease?

a) dermatitis herpetiformis

b) urticaria

c) porphyria cutanea tarda

d) vitiligo

e) alopecia areata

7. In the treatment of Type I Diabetes, which of the

following is TRUE?

a) Sulfonylureas are useful as an adjunctive therapy to

insulin

b) Most patients are adequately controlled with one

type of insulin (non-mixed) only

c) Once diagnosed with Type I DM, patients must

immediately be assessed for retinopathy

d) During periods of illness or infection, patients may

require additional insulin

e) The most common initial presentation is visual

disturbance

8. Secondary causes of hyperlipidemia include all of the

following EXCEPT:

a) obesity

b) hyperthyroidism

c) diabetes

d) nephritic syndrome

e) glucocorticoids

9. A 63-year-old woman has had a gradually enlarging goiter

for at least 10 years. She has no symptoms of

hyperthyroidism. On physical examination, both thyroid

lobes are irregular, firm and nontender. Her serum

thyroxine (T4) level is 120 nmol/L (normal range is

0.51 - 142 nmol/L) and triiodothyronine (T3) -resin uptake

is normal at 30%. The patient is treated with 50 ug of T3

daily in an attempt to decrease the size of the goiter.

Several weeks later she develops weakness,

palpitations, insomnia, and a tremor; she has lost 2.3 kg

(5 lb). The size of the thyroid gland is unchanged; her

serum T4 level is now 115 nmol/L. These findings

indicate that the patient:

a) has developed Graves' disease

b) has painless thyroiditis

c) has autonomous endogenous thyroid function

d) is taking more T3 than was prescribed

e) has had a hemorrhage into a thyroid nodule

12 – Sample Questions MCCQE 2002 Review Notes

ENDOCRINOLOGY . . . CONT.

10. In deciding upon the ideal dosage of replacement

therapy for hypothyroidism, each of the following factors

is considered EXCEPT:

a) subjective patient response

b) findings on physical examination

c) response of the radioactive iodine uptake

d) plasma TSH level

e) level of free thyroid hormone measured in the serum

11. An asymptomatic postmenopausal 54-year-old woman

sustained a slight concussion in an automobile accident.

X-rays of the skull show an enlarged sella turcica.

Computed tomography (CT scan) of the skull shows no

density within the sella turcica. This patient most likely

has:

a) A familial syndrome associated with hypercalcemia

and pheochromocytoma

b) Pigmentation of skin, buccal mucosa, and pressure

areas

c) Elevated serum calcitonin levels

d) Normal endocrine function

e) Hypogonadism with hyposomnia

12. A 32-year-old diabetic did not take his usual dose of

30 units of lente insulin and 5 units of regular insulin

because he had an "upset stomach with vomiting". The

next day he was admitted to hospital. His plasma

glucose was 29.9 mmol/L and the urine was strongly

positive for ketones. The serum bicarbonate was

6 mEq/L, arterial pH was 7.05 and serum potassium was

5.4 mEq/L. Which one the following statements is most

likely to be TRUE?

a) The predominant ketone body in the serum is

acetoacetate

b) The elevated serum potassium level reflects total

body potassium in this patient

c) The plasma glucagon level will be increased

d) The serum phosphate concentration will rise during

insulin therapy

e) The patient followed the correct course by

withholding insulin in the absence of food intake

For 13, 14, and 15.

A 32-year-old woman visits her physician because of agitation,

weight loss, and inability to sleep. When questioned

further, she reveals an increased appetite, and an increased

frequency of bowel movements. Previously, she had regular

menstrual periods, but now they are less frequent and

irregular. During the physical examination, the physician

notes that her skin is warm and moist and that she has a fine

tremor of the fingers, hyperreflexia, and lid lag. The woman

has moderately severe exophthalmos, and her upward gaze

seems weak and uncoordinated.

13. Which one of the following disease processes is most

likely manifesting itself?

a) A thyroid adenoma that is secreting thyroxine

b) Inappropriate hypothalamic secretion of TRH

c) Graves’ disease

d) Hashimoto’s disease

e) Sick euthyroid syndrome

14. All of the following statements correctly pair a useful

medication with its mechanisms of action EXCEPT

a) Propylthiouracil (PTU) blocks the coupling reaction in

T4 synthesis

b) Methimazole (MMU) reduces peripheral conversion of

T4 to T3

c) Radioactive iodine destroys follicular cells in the

thyroid

d) Propranolol blocks the sympathetic components of

thyrotoxicosis

e) Prednisone may relieve the mechanical exophthalmos

and opthalmoplegia by reducing inflammation.

15. All of the following laboratory test results are consistent

with the clinical picture EXCEPT:

a) decreased T3 resin uptake

b) decreased TSH response to a TRH challenge

c) decreased serum TSH

d) increased serum T4 concentration

e) positive test for circulating antibodies against the

TSH receptor

16. Which of the following disorders is NOT associated with

osteoporosis?

a) pheochromocytoma

b) prolactinoma

c) Cushing’s syndrome

d) Kleinfelter’s syndrome

e) Graves’ disease.

17. A 41-year-old man has been diagnosed with

panhypopituitarism. Which of the following is not part of

the appropriate management of this patient?

a) L-thyroxine

b) hydrocortisone

c) fludrocortisone

d) testosterone

e) Medic Alert bracelet

18. While not always the presenting complaint, the earliest

symptom/sign of a pituitary tumour is usually:

a) visual disturbance (e.g. bitemporal hemianopsia)

b) gonadal insufficiency

c) headache

d) extraocular muscle paresis

e) diabetes insipidus

19. An elderly diabetic woman with mild renal insufficiency

presents to the emergency room with confusion. Blood

glucose was 1.5 mmol/L. She is treated with glucose IV

for 24 hours, discharged and instructed not to take any

medications. 6 hours after discharge, she is brought back

to the emergency department in a coma. Blood glucose

is now 1 mmol/L. Which of the following hypoglycemic

agents is the patient most likely using?

a) repaglinide

b) acarbose

c) metformin

d) rosiglitazone

e) glyburide

MCCQE 2002 Review Notes Sample Questions – 13

ENDOCRINOLOGY . . . CONT.

20. A 47-year-old man is admitted to hospital to be

evaluated for hypertension and hypokalemia. After the

patient has been restricted to a 120 mEq/day Na diet for

1 week, his plasma aldosterone twice the normal level.

Which is the next MOST appropriate step?

a) fludrocortisone suppression test

b) cosyntropin (ACTH) stimulation test

c) measurement of plasma renin activity

d) CT adrenals

e) treat with spironolactone

21. Which of the following is considered diagnostic of

diabetes:

a) random plasma glucose level of 10 mmol/L

b) fasting plasma glucose level of 6.8 mmol/L

c) on OGTT, 1 hour plasma glucose level > 11.1

d) on OGTT, 2 hour plasma glucose level > 7.8

e) presence of polyuria and polydipsia and plasma

glucose level of 12 mmol/L

22. A 35-year-old man gives a history of paroxysmal

hypertension, headache, diaphoresis and palpitations.

His father has hypertension and hypercalcemia and a

paternal aunt died from a thyroid cancer. The physician

would be most likely to measure serum levels of:

a) calcitonin, calcium, and glucose

b) glucagon, insulin, and cholesterol

c) thyroid hormone, adrenomedullin, and phosphorus

d) CRH, gastrin, and renin

14 – Sample Questions MCCQE 2002 Review Notes

FAMILY MEDICINE

1. A 28 year old sexually active woman in a stable

relationship for 2 years comes to your office for an

annual check-up. Which of the following is NOT a

recommended screening tool in the Periodic Health

Exam?

a) counseling re: sun exposure

b) pap smear

c) breast self examination

d) counseling re: folic acid supplementation

e) counseling re: bicycle helmet use

2. A 26 year old woman reports suffering from moderately

severe unilateral headaches approximately twice a

month for the past year. The headaches are pulsating in

quality with associated nausea and photophobia.

Physical examination is normal. Which of the following is

NOT appropriate abortive treatment for an acute attack?

a) NSAIDS

b) sumatriptan

c) DHE

d) amitryptiline

e) ergotamine

3. A 63 year old man with a 10 year history of type 2

diabetes is screened for microalbuminuria.

Albumin:creatinine ratio is 2.6. Which of the following is

the appropriate first step in management?

a) repeat albumin:creatinine in one year

b) repeat albumin:creatinine test twice over the next

3 months and if abnormal, get 24 hour urine for

creatinine clearance

c) referral to nephrologist

d) adjust dosages of oral hypoglycemic medications

e) reduce dietary protein

4. A 38 year old businesswoman and mother of two children

presents with a 3 month history of fatigue. Past medical

history is remarkable for GDM in her last pregnancy.

Pallor noted, otherwise normal physical exam. Which of

the following investigations is NOT indicated at this time?

a) appropriate assessment for anxiety and depression

b) appropriate assessment of current life stressors,

past trauma and abuse

c) CBC

d) serum glucose

e) BUN, creatinine

5. The target LDL-C for a very high risk patient (10 year risk

CAD > 30%, history of cardiovascular disease, or

diabetes) is:

a) < 2.5 mmol/L

b) < 3.0 mmol/L

c) < 3.5 mmol/L

d) < 4.0 mmol/L

e) < 5.0 mmol/L

6. A 65 year-old newly-diagnosed hypertensive male is

about to begin anti-hypertensive medication. Which of

the following should you NOT prescribe as first-line

therapy:

a) angiotensin-converting enzyme inhibitor

b) long acting dihydropyridine

c) beta-adrenergic antagonist

d) low dose thiazide diuretic

e) high dose hydrochlorothiazide

7. A 32 year old man presents with a 2 week history of

persistent cough and generally feeling unwell. On

examination, T= 37.7 , HR= 80, BP = 120/70, RR = 16.

Which of the following is INAPPROPRIATE treatment?

a) symptomatic antipyretics

b) antitussives

c) bronchodilator bid

d) tetracycline 250 mg qid

e) rest and fluids

8. A 40 year old smoker presents with a 3 day history of

rhinorrhea, cough, and headache. No vital sign

abnormalities present. Which of the following is the

best management?

a) educate patient about duration of symptoms, suggest

hydration, analgesics and dextromethorphan prn

b) educate patient about duration of symptoms and

have her return to clinic in 2 days

c) order CXR

d) order sputum culture

e) educate patient about smoke exposure and risk of

URTI and prescribe erythromycin x 10 days

9. Which of the following is CORRECT regarding depression?

a) The lifetime risk of major depressive disorder is 55%

for women

b) Early treatment of major depressive disorder does

not improve outcomes

c) Risk of recurrence after 3 episodes of major

depressive disorder is 90%

d) Most patients will need at least two years of

pharmacological treatment

e) Most primary care patients with depression should

be referred for psychiatric consultation

10. When prescribing the oral contraceptive pill (OCP), which

of the following should be done:

a) Inform patients that the OCP protects against STDs

b) Perform breast, abdominal and pelvic exams and

assess blood pressure

c) Warn patients that their risk of dysmenorrhea is

increased

d) Perform a complete physical exam after taking a

thorough sexual history

e) Perform a Beta-HCG test prior to prescribing OCP

MCCQE 2002 Review Notes Sample Questions – 15

FAMILY MEDICINE . . . CONT.

11. Which of the following statements regarding pharyngitis

is/are TRUE:

a) The most common etiologic agent is viral

b) Bacterial causes for sore throats occur predominately

in pre-school aged children

c) Delaying treatment for Group A ß-hemolytic

Streptococcal pharyngitis by 48 hours does not

increase the risk of rheumatic fever

d) a) and c)

e) all of the above

12. In the context of low back pain, x-ray imaging should be

ordered in these cases except:

a) 25-year-old male i.v. drug user with 3 week history of

low back pain

b) 64-year old female with 48-hour history of back pain

and numbness in the right leg extending to the knee

c) Previously healthy 37-year-old male with 3 month

history of low back pain

d) 56-year-old woman with well controlled hypertension

and 4 week history of low back pain

e) 18-year-old construction worker with 1 week history

of low back pain

13. Acceptable methods of assessing for obesity include the

following but:

a) BMI

b) daily calorie intake

c) waist-hip ratio

d) percent body fat

14. Which of the following is characteristic of alcohol

dependence:

a) 23-year-old male college student who consumes an

average of 23 drinks per week and exhibits no

withdrawal symptoms

b) 65-year-old recently retired engineer who binge

drinks two times a month

c) 43-year-old married female, recently fired, and who

has been in 2 MVAs involving alcohol this past year

d) 35-year-old male former AA member who has not

drunk in 10 years

16 – Sample Questions MCCQE 2002 Review Notes

GASTROENTEROLOGY

1. Pseudomembranous colitis is usually caused by toxins of

which organism?

a) Staph aureus

b) Clostridium difficile

c) Clostridium perfringens

d) Clostridium botulinum

e) Bacteroides fragilis

2. A 13 year old presents with increased serum bilirubin

and vague nonspecific complaints. Labs show increase in

unconjugated bilirubin with normal liver function tests

and negative hepatitis screens. There is no evidence of

liver abnormality or hemolysis. The most likely diagnosis

is?

a) Gilbert’s syndrome

b) biliary atresia

c) hepatoma

d) cholecystitis

e) cholangiocarcinoma

3. After noticing his wife avidly eats clay, starch, and ice, her

husband brings her to the ER. History reveals she is a

vegetarian, and in her second trimester of pregnancy.

The most likely disease afflicting this woman is?

a) megaloblastic anemia

b) hepatolenticular disease

c) gastric carcinoma

d) pica

e) hypermethioninemia

4. The ecchymosis of the flank seen in patients with acute

pancreatitis is named after?

a) Trousseau

b) Goodpasture

c) Grey-Turner

d) Cullen

e) Wernicke-Korsakoff

5. The best test for a massive lover GI bleed is?

a) colonoscopy

b) angiography

c) radiolabelled RBC scan

d) barium enema

e) fecal occult blood

6. An elderly woman presents with blood per rectum with a

3 day history of severe ab pain, anemia, hypotension

and CAD. The most likely diagnosis is?

a) ulcerative colitis

b) crohn’s disease

c) irritable bowel disease

d) ischemic bowel (colitis)

e) infectious diarrhea

7. All are causes of splenomegaly EXCEPT:

a) infectious mononucleosis

b) thalassemia minor

c) congestive heart failure

d) sarcoidosis

e) diabetes

8. A diagnosis of Irritable bowel sydrome is based on:

a) Rome criteria which emphasizes negative features of

the disease

b) Rome criteria which emphasizes positvie features of

the disease

c) a diagnosis of not an exclusion

d) all of the above

9. Diagnosis of celiac sprue includes all of the following

EXCEPT:

a) evidence of malabsorption

b) abnormal small bowel biopsy

c) improvement in sign and symptoms with a

gluten- containing diet.

d) positive anti-endomysial antibody.

10. You suspect a patient with a fat malabsorption syndrome.

Which symptom would you NOT expect?

a) night blindness

b) metabolic bone disease

c) bleeding disorder

d) lactose intolerance

11. A 12-year-old boy is jaundiced and has an enlarged liver

on palpation. Lab values show increased ALT,

increased AST, increased conjugated bilirubin.

Kayser-Fleisher rings are noticed on the slit-lamp.

A diagnosis of Wilson’s disease is made. All of the

following are true EXCEPT:

a) the primary defect is a mutation in ceruloplasmin

b) the patient likely has increased levels of serum free

copper and liver copper and increased urinary copper

with penicillamine challenge

c) Wilson’s disease is inherited in an autosomal recessive

pattern

d) patients can demonstrate increased serum

ceruloplasmin

e) in normal patients, more than 90% of serum copper

circulates bound to ceruloplasmin

12. All of the following can be causes of conjugated

hyperbilirubinemia EXCEPT:

a) Wilson’s disease

a) Criggler-Najjar syndrome I

a) primary biliary cirrhosis

a) gallstones

a) viral hepatitis

13. The most common cause of a significant

upper GI bleed is:

a) Mallory Weiss tear

a) angiodysplasia

a) peptic ulcer

a) dieulafoy lesion

a) antritis

14. The medical treatment to be of most benefit in the

therapy of both variceal and non-variceal

upper GI bleed is:

a) octreotide

b) omprazole

c) ice water lavage

d) vasopressin

e) ranitidine

MCCQE 2002 Review Notes Sample Questions – 17

GASTROENTEROLOGY . . . CONT.

15. A 50 year old man was admitted for a bleeding duodenal

ulcer 2 months ago. He was discharged from hospital

with an appropriate course of H. Pylori eradication

treatment. He has since completed the therapy and is

currently asymptomatic. Which of the following is the

BEST non-invasive test to confirm eradition of H. Pylori:

a) urea breath test

b) endoscopy and histology

c) IgG serology

d) skin testing

16. Which one of the following statements regarding liver

failure is TRUE?

a) a liver biopsy is a not required with a good history of

alcoholism

b) encephalopathy may be reversible

c) SBP (spontaneous bacterial peritonitis) is usually a

result of blunt abdominal trauma

d) coagulopathy is a result of poor diet in most patients

with cirrhosis

e) the size a varix is proportional to the degree of

portal hypertension

18 – Sample Questions MCCQE 2002 Review Notes

GERIATRIC MEDICINE

1. Which of the following is NOT a cause of urinary

incontinence?

a) diabetes

b) coagulopathies

c) restricted mobility

d) UTIs

e) prostatic disease

2. Which of the following physiological changes affect

pharmacokinetics in the elderly?

a) increased body albumin

b) decreased body fat

c) increased GFR

d) decreased hepatic blood flow

e) increased hepatic phase I reactions

3. Which of the following statements is TRUE:

a) The ratio of males to females over 85 in Canada

is 2:1

b) Accidents are the most common cause of death in

Canadians over 65

c) During normal bereavement, symptoms of

depression should last for up to one year

d) all of the above

e) none of the above

4. The functional assessment of IADLs in the elderly

include all of the following EXCEPT:

a) food preparation

b) ability to climb stairs

c) driving

d) managing finances

e) shopping

5. Which of the following is NOT a cause of failure to thrive in

the elderly:

a) alopecia

b) poor dentition

c) depression

d) neglect

e) dementia

MCCQE 2002 Review Notes Sample Questions – 19

GYNECOLOGY

1. Which of the following is not consistent with PCOD?

a) high testosterone

b) low LH

c) average age of presentation 15-35 years

d) low or normal FSH

e) hirsutism

2. Which of the following is the most common site of

occurrence for endometriosis?

a) broad ligament

b) uterosacral ligaments

c) ovary

d) rectosigmoid colon

e) appendix

f) lung

3. Which of the following are diagnostic characteristics of

endometriosis? (Multiple answers)

a) constant pelvic pain

b) chocolate cysts

c) deep dyspareunia

d) blueberry spots

e) cyclic dysmenorrhea

f) infertility

g) bladder symptoms

h) bowel symptoms

i) powder burn lesions

4. Which of the following are considered risk factors for

ectopic pregnancy? (Multiple answers)

a) alcohol consumption

b) previous PID

c) use of IUD

d) appendectomy

e) previous ectopic pregnancy

f) use of assisted reproductive techniques

g) grand multiparity

h) endometriosis

5. Which of the following are considered predisposing

factors for candidiasis? (Multiple answers)

a) diabetes

b) pregnancy

c) HIV

d) organ transplant recipient

e) BCP

f) use of a diaphragm

g) antibiotic therapy

h) hypertention

i) cigarette smoking

6. Regarding condylomata/genital warts which of the

following is/are TRUE? (Multiple answers)

a) spread can be prevented using condoms

b) infection can be latent, only detectable with DNA

hybridization

c) some viral serotypes are associated with an

increased incidence of cervical CA

d) removal of lesions is proven to decrease recurrence

e) lesions present during pregnancy is an absolute

indication for C/S

f) treatment is through either physical or chemical

modalities

7. Which of the following is correct regarding the presentation

and treatment of genital herpes? (Multiple answers)

a) inguinal lymphadenopathy is present with first

infection

b) recurrent infections are usually less severe and less

frequent

c) presentation is four to six months after exposure

d) initial presentation is as a painless chancre on vulva

e) diagnosis is made on viral culture

f) treatment with acyclovir is curative

g) outbreak is usually preceded by a tingling, burning

prodrome

8. Which of the following are absolutely necessary for the

diagnosis of PID? (Multiple answers)

a) temperature > 39.0 C

b) adnexal tenderness

c) cervical motion tenderness

d) elevated white count

e) positive culture for N. gonorrhea, or C. Trachomatis

f) elevated ESR

g) lower abdominal pain

9. Regarding PAP smear screening protocol, which of the

following are recommended? (Multiple answers)

a) should be performed yearly from onset of sexual

activity until age 69

b) requires only an endocervical cell sampling

c) is equally effective in identifying both SCC and

adenocarcinoma

d) a patient can be released from screening after two

consecutive negative tests

e) after three negative tests, screening intervals can be

increased up to 3 years

f) an inadequate sample requires a repeat PAP within

3 months

20 – Sample Questions MCCQE 2002 Review Notes

GYNECOLOGY . . . CONT.

10. Which of the following are consistent with the clinical

presentation of fibroids? (Multiple answers)

a) menorrhagia

b) abnormal bleeding pattern

c) abdominal heaviness

d) increased abdominal girth

e) amenorrhea

f) infertility

g) abdominal pain

h) difficulty emptying bladder

i) difficulty defecating

11. Non-contraceptive benefits of oral contraception pills

include:

a) Reduced benign breast disease and ovarian cysts

b) Reduced dysmenorrhea

c) Reduced anemia

d) Reduced risk of ovarian carcinoma

e) All of the above

12. Regarding infertility:

a) It is defined as failure to conceive after one year of

regular unprotected intercourse

b) It occurs in approximately 50-60% of couples

c) Most cases are due to unknown factors

d) A normal sperm count from semen analysis is

> 5 million sperm/mL

e) A hysterosalpingogram plays no role in potential

evaluation of infertility

13. A 20 year old G0 woman presents to your office with a

4-month history of amenorrhea. She had previously

normal and regular cycles since age 12 years. Your initial

evaluation of this problem includes:

a) Thorough history and physical examination

b) A beta-hCG test

c) TSH and prolactin levels

d) All of the above

e) none of the above

14. Absolute contraindications to HRT include:

a) undiagnosed vaginal bleeding

b) known or suspected cancer of the ovary

c) acute renal disease

d) obesity

e) none of the above

15. Which of the following condition(s) are associated with

Polycystic Ovarian Syndrome?

a) obesity

b) hirsutism

c) insulin resistance

d) acanthosis nigricans

e) All of the above

MCCQE 2002 Review Notes Sample Questions – 21

HEMATOLOGY

1. You would expect to find an increased bleeding time in

all of the following conditions EXCEPT:

a) Osler-Weber-Rendu

b) HSP

c) ITP

d) Celiac disease

e) Splenomegaly

2. All of the following characterize chronic ITP EXCEPT:

a) onset usually following recent viral infection

b) spontaneous remission uncommon

c) peak age of onset 20-40 years

d) occurs more frequently in women

e) insidious onset of bleed

3. Which of the following coagulation factors is NOT

effected by chronic liver disease

a) factor VII

b) factor V

c) fibrinogen

d) factor VIII

e) factor IX

4. What disorder results in prolonged bleeding time and

decreased factor VIII coagulation activity?

a) Hemophilia A

b) Hemophilia B

c) von Willebrand’s disease

d) Vitamin K deficiency

e) None of the above

5. A young girl with thalassemia is undergoing a blood

transfusion and suddenly develops back pain, fever and

becomes tachypneic. The likely diagnosis is:

a) Delayed hemolytic transfusion reaction

b) Febrile non-hemolytic transfusion recaction

c) Acute hemolytic transfusion reaction

d) Circulatory overload

e) Graft vs. host disease

6. Which of the following is not accurate for iron stores:

a) Serum Fe

b) Transferrin

c) Ferritin

d) Hemosiderin

e) TIBC

7. The most common cause of cold antibodies other than

idiopathic is:

a) Drug induced

b) Routine detection

c) Secondary to infection

d) Secondary to lymphoproliferative disorder

e) Autoimmune disease

8. Which of the following blood smear results are most

likely to indicate G6PD deficiency?

a) Rouleaux

b) Howell-Jolly bodies

c) Basophilic stipling

d) Heinz bodies

e) Tear drop cells

9. All of the following may be seen in aplastic anemia

EXCEPT:

a) Anemia

b) Thrombocytopenia

c) Leukopenia

d) Splenomegaly

e) increase frequency of infections

10. The cause of beta thalasemia minor is:

a) Increase HbA2 production

b) Reduced B chain production

c) Increased HbF production

d) No B chain production

e) Increased B chain production

11. Metabolic disturbances that may be seen in acute

myelogenous leukemia include:

a) high uric acid, high calcium, high magnesium

b) high uric acid, low calcium, low magnesium

c) high uric acid, high calcium, high phosphate

d) high uric acid, low calcium, low phosphate

e) low uric acid, high calcium, high phosphate

12. Clinical features of chronic myelogenous leukemia include

all EXCEPT:

a) left upper quadrant pain and fullness

b) pruritus

c) peptic ulcers

d) priapism

e) jaundice

13. The MOST COMMON presentation of essential

thrombocythemia is:

a) weight loss, fever

b) bleeding

c) thrombosis

d) asymptomatic

e) splenomegaly

14. A 22-year old male student presents with enlarged cervical

lymph nodes that become painful after alcohol

consumption. The only remarkable feature on history is an

8 lb weight loss over 2 months. The most likely diagnosis

would be:

a) ALL

b) Hodgkin’s lymphoma

c) Non-Hodgkin’s lymphoma

d) CLL

e) multiple myeloma

15. Possible treatments for multiple myeloma or its

complications include all EXCEPT:

a) corticosteroids

b) bisphosphonates

c) radiation therapy for bone lesions

d) renal transplant

e) plasmapheresis

22 – Sample Questions MCCQE 2002 Review Notes

INFECTIOUS DISEASES

1. Group A Strep is least likely to cause which of the

following complications:

a) scarlet fever

b) necrotizing faciitis

c) impetigo

d) subacute bacterial endocarditis

e) glomerulonephritis

2. Septra is used in AIDS patients to prevent which

opportunistic organism?

a) Pneumocystis carinii

b) M. tuberculosis

c) S. pneumoniae

d) CMV

e) Cryptococcus

3. Severe bloody, afebrile diarrhea is associated with what

bacterial infection?

a) Salmonella infection

b) Enteroinvasive E.coli infection

c) Enterohemorrhagic E. coli infection

d) Enterotoxigenic E. coli infection

e) Giardiasis

4. Human Herpes Virus 6 is associated with what disease?

a) Infectious mononucleosis

b) CMV infection

c) Herpes Simplex infection

d) Roseola

e) Chicken Pox

5. Cat’s scratch disease is caused by what organism?

a) Leishmania spp.

b) Bartonella henselae

c) Treponema pallidum

d) Toxoplasmosa gondii

e) Ancylostoma braziliensei

6. Predictors of HIV progression include the following

EXCEPT:

a) CD4 cell count

b) plasma HIV RNA levels at set point

c) onset of HIV related symptoms

d) age at initial infection

e) method of transmission (i.e. anal intercourse versus

vaginal intercourse)

7. Protozoa are associated with all of the following except

a) unicellular organism

b) produce larvae

c) do not cause eosinophilia

d) indefinite lifespan

e) mulitply within host

8. Inwhich disease would Donovan bodies be present?

a) genital warts

b) lymphogranuloma venereum

c) syphilis

d) granuloma inguinale

e) chancroid

9. Which disease is NOT caused by Chlamydia sp.?

a) lymphogranuloma venereum

b) pelvic inflammatory disease

c) granuloma inguinale

d) inclusion conjunctivitis

e) nongonococcal urethritis

10. If a patient comes to your office complaining of chest pain

and a cough producing a current jelly-like sputum, which

organism would be the most likely cause of his/her

pneumonia?

a) Streptococcus pneumoniae

b) Haemophilus influenzae

c) Pseudomons aeruginosa

d) Influenza virus

e) Klebsiella pneumoniae

11. Which organ in the human body is the most sensitive to

disseminated intravascular coagulation in a case of

meningitis?

a) thyroid

b) adrenal

c) lateral ventricle of the brain

d) spleen

e) aorta

12. The Negri body is a pathogonomiic sign of:

a) acute bacterial meningitis

b) arbovirus encephalitis

c) rabies

d) polio

e) chronic meningitis

13. If untreated, this infection can lead to cholangiocarcinoma.

a) Schistosomiasis

b) Clonorchis sinesis

c) Strongyloidosis

d) Trypanosoma

e) E. coli

14. A 10 year old girl from El Salvador is brought to the

pediatric outpatient clinic. On the growth charts, she is

< 1 percentile. A CBC reveals a profound anemia. She has

moderate edema of the lower extremities and face. Stool

is sent for ova and parasites (O&P), and the lab reports the

presence of helminth eggs. The most likely parasite

compatible with her clinical presentation is?

a) hookworm

b) Strongyloidosis

c) pinworm

d) pork tapeworm

15. A 48 year old make who received a bone marrow

transplant 6 months previously for chronic myelogenous

leukemia is admitted with wedge-shaped infiltrates on

chest x-ray. He is experiencing severe respiratory difficulty

and is coughing up blood. Despite aggressive treatment,

he dies shortly after admission. At autopsy, pulmonary

infarcts are found. The pulmonary vessels contain fungal

hyphae with acute angle branching. The most likely

diagnosis in infection with:

a) Asperigillus spp.

b) Candida spp.

c) Blastomycosis

d) Pneumocystis carinii

MCCQE 2002 Review Notes Sample Questions – 23

NEPHROLOGY

1. A 63 year old woman with a long history of chronic renal

failure is seen because of fatigue. She gets tired after

walking 4 blocks, but is comfortable at rest. Her only

medication is a calcium channel blocker for

hypertension. There is no history suggestive of bleeding.

Her physical examination reveals pallor but no other

abnormalities. Lab investigations reveal that a

Hgb = 85 g/L, MCV = 88 fL, ferritin = 210 ug/L,

vitamin B12 = 210 pmol/L, a normal RBC folate, and

serum creatinine of 379 umol/L. Two months ago, her

Hgb level was 90 g/L. The MOST APPROPRIATE

THERAPY for her anemia AT THIS POINT is:

a) administration of erythropoietin

b) dialysis

c) renal transplantation

d) vitamin B12 injections

e) oral iron

2. A 22 year old woman with a history of type 1 diabetes

mellitus is brought to the emergency room in coma.

Blood tests indicate a wide anion gap metabolic

acidosis. If her metabolic acidosis is due solely to

diabetic ketoacidosis, which of the following findings is

NOT consistent:

a) blood glucose increased

b) serum ketones increased

c) serum osmolar gap increased

d) extracellular fluid volume reduced

e) urine dip positive for ketones

3. A patient in acute renal failure comes to the emergency

room. His serum potassium value is 8.0 mM. Of the

following ECG changes listed below, which ONE is NOT

consistent with hyperkalemia?

a) wide QRS complexes

b) peaked T waves

c) peaked P waves

d) ventricular fibrillation

e) prolonged PR interval

4. A chronic dialysis patient who passes no urine has missed

3 hemodialysis treatments in the last week and comes into

the emergency room with a serum potassium value of

8.5 mmol/L. An ECG shows cardiac changes consistent with

severe hyperkalemia. Blood glucose is 4.5 mmol/L. Which

ONE of the following would be the best action to take:

a) Give the patient a prescription for an oral potassium

exchange resin and arrange dialysis within 24 hours

b) Give insulin and glucose intravenously

c) Give insulin and calcium gluconate intravenously and

inhaled salbutamol

d) Give insulin, calcium gluconate and glucose

intravenously and arrange for urgent dialysis

e) Give inhaled salbutamol and larger doses of

intravenous furosemide

5. Which of the following is the LEAST APPROPRIATE

indication for dialysis in the setting of acute renal failure?

a) extracellular fluid volume overload unresponsive to

diuretics

b) hyperkalemia unresponsive to medical attempts to

lower the plasma potassium level

c) plasma creatinine greater than 300 umol/L

in an otherwise asymptomatic patient

d) uremic pericarditis

e) uremic encephalopathy with seizures

6. The COMMONEST cause of acute renal failure in

hospitalized patients is:

a) acute crescentic glomerulonephritis

b) acute tubular necrosis (ATN)

c) cyclosporine nephropathy

d) HIV nephropathy

e) obstructive uropathy

7. The COMMONEST cause of death in dialysis and renal

transplant patients is:

a) cardiovascular disease

b) HIV-associated complications from repeated

infusions of blood products

c) infection

d) neoplasia

e) suicide

8. A 21 year old woman is referred for edema of the ankles

that developed 3 weeks earlier. Physical examination

shows moderate bilateral ankle edema. The chest was

clear and heart sounds were normal. Jugular venous

pressure was 1 cm above the sternal angle. Urinalysis

shows 3+ protein and occasional red and white blood

cells. Plasma creatinine was normal and plasma albumin

was reduced at 36 g/L.

Which of the following is the LEAST likely renal

diagnosis?

a) acute tubular necrosis (ATN)

b) membranous glomerulonephritis

c) minimal change glomerulonephritis

d) lupus nephritis

e) post-infectious glomerulonephritis

9. A 66 year old man is referred to you for hypertension

which was not noted until this year, despite yearly

physical examinations for many years. Apart from recent

headaches he has no complaints and was not taking any

medications. Blood pressure in your office was

175/100 mm Hg with a pulse of 80/min. Examination of

the optic fundi showed arteriolar narrowing. Bilateral

femoral bruits were present. Urinalysis showed 1+

protein and granular casts. Plasma creatinine was

210 umol/L. The MOST LIKELY diagnosis is:

a) atheromatous emboli

b) bilateral renal artery stenosis

c) essential hypertension

d) pheochromocytoma

e) proliferative glomerulonephritis

24 – Sample Questions MCCQE 2002 Review Notes

NEPHROLOGY . . . CONT.

10. A 36 year old man presents with ankle edema and an

elevated jugular venous pressure. Plasma creatinine is

350 umol/L. Past history reveals a polyarthritis for

6 months treated with ibuprofen and allergic rhinitis.

Physical exam reveals several active joints in both hands

and an itchy skin rash over the trunk. BP = 170/100 mm Hg

and he has moderate peripheral edema. Urinalysis shows

a trace of protein, trace blood and on microscopy many

white blood cells and white blood cell casts are seen.

The MOST LIKELY diagnosis is:

a) atheromatous emboli

b) interstitial nephritis

c) lupus nephritis

d) membranous glomerulonephritis

e) renal artery stenosis

11. Acute diffuse proliferative glomerulonephritis is usually

accompanied by each of the following findings EXCEPT:

a) red blood cell casts on microscopic urinalysis

b) proteinuria

c) pigmented casts on microscopic urinalysis

d) blood clots in the urine

e) none of the above

12. The anion gap is increased in metabolic acidosis

associated with each of the following EXCEPT:

a) diabetic ketoacidosis

b) renal tubular acidosis

c) acute tubular necrosis

d) ethylene glycol intoxication

e) lactic acidosis

13. Treatment of hypertension caused by bilateral renal

artery stenosis should NOT include which of the

following?

a) weight reduction if obese

b) drug therapy with angiotensin-converting enzyme

inhibitors

c) percutaneous transluminal renal angioplasty

d) surgical renal revascularization

e) sodium restriction

14. Hyperkalemia is reduced by each of the following

EXCEPT:

a) administration of calcium gluconate intravenously

b) administration of glucose, insulin and sodium

bicarbonate intravenously

c) oral administration of potassium-exchange resins

with sorbitol

d) hemodialysis with low potassium dialysate

15. The anemia of chronic renal failure is usually due to which

of the following?

a) malabsorption of iron

b) vitamin B12 deficiency

c) blood loss

d) decreased erythropoietin production

e) associated inflammatory conditions

NEUROLOGY

1. 32 year old woman with right-hand pain waking her up at

night. Physical exam reveals mild weakness of right

thumb adduction and some thenar wasting. What is the

most likely diagnosis?

a) cervical radiculopathy

b) carpal tunnel syndrome

c) tendinits

d) syringomyelia

2. 35 year old woman with one week history of bilateral leg

weakness. Physical exam reveals moderate leg weakness,

mild arm weakness, and decreased reflexes. What is the

most likely diagnosis?

a) Lyme disease

b) Parkinson's disease

c) Multiple Sclerosis (MS)

d) Guillian-Barré syndrome

3. Patient with mild left-sided hearing loss and absent left

corneal reflex. Where is the lesion?

a) left cerebellar pontine angle

b) lateral medulla

c) left CN V

d) left cochlea

4. 75 year old man with sudden onset of severe leg

weakness. Physical exam reveals bilateral leg weakness,

loss of pain and temperature sensation with intact

vibration and positon sense in both legs. What is the

likely diagnosis?

a) anterior cerebral artery territory stroke

b) lumbar radiculopathy

c) anterior spinal artery occlusion

d) diabetic polyneuropathy

5. Which of the following is NOT consistent with

amyotrophic lateral sclerosis (ALS)?

a) distal arm and leg weakness

b) coexistence of both upper and motor neuron signs

c) slowed motor nerve conduction velocities

d) fasciculations

6. 52 year old woman with incontinence and “dizzy spells.”

Physical exam reveals rigidity, bradykinesia, postural

instability, mild ataxia, and postural hypotension. What is

the likely diagnosis?

a) vitamin B12 deficiency

b) Parkinson’s disease

c) normal pressure hydrocephalus

d) multiple systems atrophy

7. Patient with dizziness, left arm and leg weakness, loss of

pain and temperature senasation in the left face and

right extremeties. What is the likely diagnosis?

a) lateral medullary syndrome

b) migraine with aura

c) middle cerebellar artery territory stroke

d) benign paroxysmal positional vertigo (BPPV)

8. Choose the ONE CORRECT statement. A lesion of the left

cerebellum could cause?

a) akinesia

b) right arm and leg ataxia

c) left arm and leg weakness

d) spasticity of the right arm and leg

e) dysmetria on finger to nose testing of the left arm

9. The following statements concerning the pathophysiology

of Parkinson’s disease are correct, EXCEPT:

a) symptoms appear after there has been a substantial

loss of the pigmented neurons of the substantia nigra

b) the internal segment of the globus is the major outflow

center of the basal ganglia

c) the globus pallidus interna is normally excitatory

to the thalamus

d) a pallidotomy, a surgical lesion placed in the globus

pallidus is helpful in some patients with Parkinson’s

disease

e) dopaminergic neurons of the substantia nigra project

to the putamen

10. Choose the ONE CORRECT statement regarding Wernicke’s

aphasia:

a) patients with Wernicke’s aphasia have dysprosodic

speech

b) weakness of the face and arm are commonly found in

patients with Wernicke’s aphasia

c) the speech of patients with Wernicke’s aphasia is fluent

with normal or slightly increased speed

d) patients with Wernicke’s aphasia almost universally

fail to understand the commance “take off your

glasses”

e) patients with Wernicke’s aphasia cannot write but

they are still able to read

11. All of the following statements regarding aphasia are

correct, EXCEPT:

a) the left hemisphere is dominant for language even in

many left handed patients

b) patients with Broca’s aphasia often have upper motor

weakness of the right arm and face

c) patients with Broca’s Conduction, and Wernicke’s

aphasias all have difficulty with repetition

d) both Wernicke’s and Broca’s areas are supplied by the

middle cerebral artery

e) patients with transcortical type aphasias have a lesion

which damages both Wernicke’s and Broca’s cortical

areas

12. A patient who is unable to use a comb but has no muscle

weakness is likely to have a lesion in which part of the

central nervous system?

a) the hand area of the motor cortex

b) the vermis of the cerebellum

c) Broca’s area

d) part of the premotor or prefrontal cortex

e) posterior parietal cortex

MCCQE 2002 Review Notes Sample Questions – 25

26 – Sample Questions MCCQE 2002 Review Notes

NEUROSURGERY

1. Which of the following is FALSE regarding intracranial

dynamics?

a) normal intracranial pressure is 6-15 mmHg

(8-18 cm H2O)

b) the relationship between an expanding intracranial

mass and the resultant rise in intracranial pressure

is linear

c) cerebral blood flow depends on cerebral perfusion

pressure and cerebral vascular resistance

d) lumbar puncture is contraindicated in patients with

known or suspected intracranial mass lesions

2. Which of the following is NOT a classic finding in

someone with raised intracranial pressure?

a) headache with nausea and vomiting

b) respiratory changes

c) increased blood pressure

d) tachycardia

3. An obsese 30 year-old woman presents with headache

and nausea, and a bilateral decrease in visual acuity.

She has no recent history of trauma and is otherwise

well, but takes tetracycline for acne. CT and MRI scans of

her head with and without contrast are normal. Of the

following options, what is the most likely diagnosis?

a) normal pressure hydrocephalus

b) benign intracranial hypertension

c) meningioma

d) acute subdural hematoma

4. What is the most common type of primary brain tumour

in adults?

a) astrocytoma

b) medulloblastoma

c) meningioma

d) vestibular schwannoma

5. Which of the following is NOT a characteristic CT feature

of a brain abscess?

a) perilesional hypodensity

b) central hypodensity

c) homogenous contrast enhancement

d) ring enhancement with contrast

6. A 45 year-old man presents with headache. On

examination, you notice that he has a large chin and

brow, as well as large clammy hands. MRI of his head

reveals a large midline suprasellar mass. What visual

deficit might you expect in this gentleman?

a) right monocular blindness

b) amaurosis fugax

c) left homonymous hemiaopia

d) bitemporal hemianopia

7. What is the most appropriat initial investigation in the

diagnosis of subarachnoid hemorrhage?

a) lumbar puncture

b) CT without contrast

c) MRI

d) CT with contrast

8. Which of the following is NOT part of the management

of vasopasm following subarachnoid hemorrhage?

a) maintaining blood pressure at or below 120/80

b) nimodipine

c) IV solutions to decrease hematocrit levels

d) angioplasty

9. A 70 year-old man complains of trouble walking. On

examination, you find that he has left leg weakness and

hyperreflexia, but relatively normal power and reflexes in

his right leg. Furthermore he has decreased position and

vibration sense in his left leg relative to his right, but

decreased pin prick sensation in his right leg relative to

his left. MRI of his lumbar spine reveals a tumour

compressing the man’s spinal cord. Where is the tumour?

a) lateral to the right side of the lumbar spinal cord

b) lateral to the left side of the lumbar spinal cord

c) within the centre of the lumbar spinal cord

d) anterior and central to the lumbar spinal cord

10. A 29 year-old bodybuilder presents complaining of right

arm pain and weakness. Examination reveals a

decreased triceps reflex on the right and decreased

sensation on the right middle finger. MRI shows a right

posterolateral intervertebral disc hernation. Where is the

herniation ?

a) C4-5 interspace

b) C5-6 interspace

c) C6-7 interspace

d) C7-T1 interspace

11. What is the most common cause of an epidural

hematoma?

a) ruptured middle cerebral atery

b) ruptured anterior cerebral artery

c) ruptured posterior communicating artery

d) ruptured middle meningeal arter

12. What is the typical appearance of an acute subdural

hematoma on noncontrast CT?

a) diffuse intraparenchymal hypodense mass

b) hyperdense biconvex mass

c) hypodense intraventricular mass

d) hyperdense concave mass

13. Which of the following is NOT a classic finding in someone

with carpal tunnel syndrome?

a) positive Tinel’s syndrome

b) hand pain sometimes awakening patient at night

c) hypothenar muscle wasting

d) positive Phalen’s sign

14. What is the most common location for pediatric brain

tumours?

a) anterior cranial fossa

b) posterior cranial fossa

c) middle cranial fossa

d) sella turcica

15. What is the term for the group of hindbrain abnormalities

involving some degree of cerebellar hernation or

hypoplasia?

a) Chiari malformation

b) Dandy-Walker malformation

c) craniosynostosis

d) myelomeningocele

MCCQE 2002 Review Notes Sample Questions – 27

OBSTETRICS

1. Polyhydramnios is associated with all of these EXCEPT:

a) premature labour

b) cord prolapse

c) postpartum hemorrhage

d) Potter’s syndrome

e) trisomy 18

2. With regard to thromboembolism and pregnancy all of

the following are true EXCEPT:

a) Risk increases with maternal age

b) Warfarin is teratogenic

c) There is an increase in factors I, VII, VIII, IX, X, XII

d) Heparin crosses the placenta

e) The uterus compresses veins contributing the

venous stasis

3. Which of the following are TRUE with regard to normal

labour:

a) Labour commences with onset of regular painful

contractions in the presence of a dilated cervix

b) Retraction of the head during the second stage

suggests an undiagnosed malposition

c) The average time for the second stage is one hour in

the multiparous woman

d) The second stage includes both a latent and active

stage

e) Signs of placental separation include a gush of bright

red placental blood

4. Which of the following is associated with profuse painless

vaginal bleeding in the third trimester?

a) abruptio placenta

b) umbilical cord prolapse

c) degenerating fibroid

d) placental insufficiency

e) placenta previa

5. Which of the following is NOT a prerequisite to the use of

forceps in obstetrical delivery?

a) patient must be fully dilated

b) full bladder

c) adequate anaesthesia

d) position of fetus known

e) ruptured membranes

6. The most common cause of postpartum hemorrhage is:

a) retained placenta

b) uterine atony

c) vaginal lacerations

d) cervical lacerations

e) uterine fibroids

7. Which of the following is NOT associated with severe

preeclampsia?

a) elevated liver transaminases

b) thrombocytopenia

c) blood pressure greater than 160/110

d) oliguria

e) hematemesis

8. A 28 year old G1P0 woman at 35 weeks presents to the

labour floor with painless vaginal bleeding. Which of the

following should NOT be done?

a) vaginal examination

b) complete blood count

c) crossmatch blood

d) ultrasound

e) fetal heart rate monitor

9. Which of the following is TRUE regarding malpresentation

of a fetus?

a) It is associated with an increased risk of congenital

anomaly

b) Compound presentation precludes a vaginal delivery

c) It commonly occurs in association with an anthropoid

pelvis

d) It may be corrected during labour by intravenous

oxytocic agents

e) It should be treated by immediate stabilizing

induction if the membranes rupture

10. All of the following are true about the renal system in

pregnancy EXCEPT:

a) The GFR is 60% greater than normal by 12 weeks

b) There is a link between UTI and low birth weight

c) Acute glomerulonephritis is a rare condition and is

usually diagnosed as preeclampsia

d) Asymptomatic bacteriuria has a frequency of 5%

e) The creatinine and urea decrease in the normal

woman compared with non-pregnant values

11. The diagnosis of pregnancy is related to which of the

following?

a) May be delayed as there is commonly a small

amount of blood loss at the time of the first missed

period

b) Nausea and vomiting usually precede amenorrhea

c) Portable doppler machine may detect a fetal heart at

6 weeks

d) In the absence of biophysical and biochemical

diagnostic aids a bimanual examination to detect

Hegar’s sign should be performed

e) The beta subunit of HCG is similar to corresponding

units on LH,FSH and TSH giving false positive results

on immunosorbent assays

12. All of the following are prerequisites for labour

suppression (tocolysis) EXCEPT:

a) intact membranes

b) live fetus

c) absence of fetal distress

d) no dilatation

e) necessary personnel if tocolysis fails

13. With regards to fetal heart monitoring, which of the

following is TRUE:

a) Early decelerations are related to a vagal response

to head compression

b) An acceleration is defined as an increase of at least

20 bpm lasting at least 20 seconds

c) Normal range is 100-180 bpm

d) A variable deceleration is the least common change

seen during labour

e) A late deceleration peaks at the same time as the

uterine contraction

14. Which of the following is NOT true:

a) With rubella infection, the greatest risk to the fetus

occurs in the first trimester

b) Genital herpes lesions is an indication for a caesarean

section

c) AZT decreases the incidence of vertical transmission

of HIV

d) Varicella vaccine is safe during pregnancy

e) All women should be screened for Hepatitis B

15. Which of the following is TRUE:

a) The puerperium refers to the first 2 weeks after

delivery

b) The uterus should reach the non-pregnant state with

in 1-2 weeks

c) Foul smelling lochia suggests endometritis

d) Lochia changes in time from lochia rubra to lochia

serosa to lochia alba

e) Postpartum blues are rare

OPHTHALMOLOGY

1. Which of the following is NOT a contraindication to pupil

dilation:

a) narrow anterior chamber

b) iritis

c) neurologic abnormality requiring pupillary

evaluation

d) iris supported anterior chamber lens implant

2. In myopics, the eyeball is:

a) too long

b) too short

c) non-spherical

d) crooked

3. The most common cause of exophthalmos in children is:

a) hyperthyroidism

b) orbital cellulitis

c) orbital tumours

d) orbital hemorrhage

4. Eversion of the lower lid margins is termed:

a) trichiasis

b) entropion

c) ectropion

d) chalazion

5. The most common type of lid carcinoma is:

a) adenocarcinoma

b) squamous cell carcinoma

c) sebaceous cell carcinoma

d) basal cell carcinoma

6. Which conjunctivitis typically begins as unilateral and

progresses to the opposite eye:

a) bacterial

b) viral

c) allergic

d) chlamydial

7. Which of the following is associated with contact lens

wear:

a) vernal conjunctivitis

b) iritis

c) scleromalacia perforans

d) giant papillary conjunctivitis

8. Which of the following should never be given for corneal

abrasions:

a) topical analgesics

b) topical antibiotics

c) pressure patch

d) topical cycloplegics

9. Dendritic lesions are characteristic of:

a) herpes zoster keratitis

b) arcus senilus

c) herpes simplex keratitis

d) scleritis

10. Severe photophobia is most characteristic of:

a) acute glaucoma

b) cataracts

c) iritis

d) corneal abrasion

11. Which is NOT a management step in iritis:

a) pressure patch

b) pupil dilation

c) systemic analgesics

d) medical work-up to determine etiology

12. Cataracts can be due to all of the following but:

a) aging

b) diabetes mellitus

c) increased lipids

d) uveitis

13. Which of the following require the most urgent attention:

a) central retinal vein occlusion

b) cataracts

c) scleritis

d) central retinal artery occlusion

14. Management for primary angle closure galucoma

includes all of the following except:

a) laser iridotomy

b) mydriatic drops

c) topical beta-blockers

d) IV hypertonic mannitol

e) systemic carbonic anhydrase inhibitors

15. Which of the following is NOT associated with primary

angle closure glaucoma:

a) steroid use

b) nausea and vomiting

c) hyperopia

d) painful red eye

e) opacified cornea

16. Which of the following is NOT a cause of a relative

afferent pupillary defect:

a) multiple sclerosis

b) optic neuritis

c) dense cataract

d) large retinal detachment

e) central retinal vein occlusion

17. Which of the following is associated with impaired pupil

dilation:

a) sympathetic stimulation

b) Adie’s tonic pupil

c) parasympathetic understimulation

d) Horner’s syndrome

18. The most common ocular infection in HIV is:

a) herples simplex

b) pnuemocystis carinni

c) cytomegalovirus

d) candida

e) toxoplasmosis

28 – Sample Questions MCCQE 2002 Review Notes

MCCQE 2002 Review Notes Sample Questions – 29

OPHTHALMOLOGY . . . CONT.

19. In a newly diagnosed patient with type 2 diabetes,

appropriate screening for retinopathy should involve:

a) begin screening five years after diagnosis

b) repeat in four years and thereafter annually

c) begin screening three years after diagnosis, then

repeat annually

d) begin screening after age 50; earlier if poor glycemic

control

20. Non-proliferative changes in diabetic retinopathy include

all of the following EXCEPT:

a) microaneurysms

b) retinal edema

c) intraretinal microvascular anomalies

d) retinal edema

e) dot and blot hemorrhages

21. Grade 5 involvement in graves disease refers to:

a) corneal involvement

b) soft tissue involvement

c) sight loss due to optic neuropathy

d) proptosis

22. Which of the following tests for a phoria:

a) Hirschberg test

b) cover test

c) cover-uncover test

d) holler test

e) accomadation reflex

23. The following are true in the management of chemical

burns to the eye EXCEPT:

a) alkali burns have a worse prognosis than acid burns

b) an alkali burn should be neutralized immediately

c) an IV drip with water should be set-up for irrigation in

the emergency room

d) cyclopegic drops should be administered

24. In differentiating the causes of red eye, photphobia is

most characteristic of:

a) cataract

b) conjunctivitis

c) angle closure keratitis

d) acute glaucoma

e) acute iritis

ORTHOPEDICS

1. A 35 year old male manual labourer sustained a

displaced subcapital hip fracture after falling at work.

The fracture was reduced and fixed with 3 cannulated

screws. Five months after the operation, he presents to

your clinic with worsening hip pain. What is the most

likely diagnosis?

a) Nonunion

b) Osteonecrosis

c) Loosening of the cannulated screws

d) Malunion

e) Osteomyelitis

2. A 25 year old professional basket ball player lands on his

right foot while it is in a pronated and externally rotated

position. He hears a “snap” and is unable to weight bear

on his right foot. An ankle X-ray reveals significant talar

shift without a visible fibular fracture. The most

appropriate next step is:

a) Closed reduction of the ankle mortise followed by

application of a cast

b) Bedrest for 4 weeks followed by intense

physiotherapy

c) X-ray the right knee to rule out a Maisonneuve

fracture

d) Open reduction with internal fixation of the right

ankle

e) Splint ankle and encourage weight bearing

3. A 75 year old lady slips on a throw rug in her living room

and falls. An X-ray reveals a displaced subcapital hip

fracture. Prior to her fall, the patient lived alone,

per formed all ADLs independently and enjoyed golfing.

The most appropriate management of this fracture is:

a) Moore’s unipolar hemiarthroplasty

b) Bedrest for 6 weeks

c) Reduction with internal fixation using 3 cannulated

screws

d) Bipolar hemiarthroplasty

e) Total hip replacement

4. A 21 year old presents to your office after injuring her

knee in a soccer game. She states that the knee clicks

when she walks and has “locked” on several occasions.

On exam there is an effusion and the knee is grossly

stable. The most likely diagnosis is:

a) Anterior cruciate ligament tear

b) Meniscal tear

c) Osteoarthritis

d) Bursitis

e) Medial collateral ligament tear

5. All of the following regarding Achilles tendon rupture are

true, EXCEPT:

a) Positive Thompson’s test

b) Palpable gap over Achilles tendon

c) Weak plantar flexion

d) May occur secondary to steroid injection

e) Treat by casting foot in dorsiflexion

30 – Sample Questions MCCQE 2002 Review Notes

MCCQE 2002 Review Notes Sample Questions – 31

OTOLARYNGOLOGY

1. Regarding laryngeal cancer, which of the following is

FALSE?

a) hoarseness appears early

b) involved nodes are not palpable in 35% of cases

c) distant metastasis appears early

d) direct extension is common

e) it is 90% five-year curable when limited to one cord

2. All of the following are removed in radical neck

dissection EXCEPT:

a) sternocleidomastoid muscle

b) external carotid artery

c) internal jugular vein

d) spinal accessory nerve

e) submaxillary gland

3. Which of the following factors is NOT associated with

squamous cell carcinoma of the larynx?

a) male sex

b) age in fifth and sixth decades

c) history of woodworking

d) large ethanol intake

e) tobacco smoking

4. In LeFort I fractures, the fragment consists of all of the

following EXCEPT:

a) upper teeth and palate

b) lower portions of the pterygoid processes

c) portions of the walls of both maxillary antra

d) nasal spine

e) bridge of the nose

5. In general, traumatic perforations of the tympanic

membrane:

a) are a surgical emergency

b) will heal spontaneously in most cases

c) usually require operative repair

d) require microsurgical repair

e) require a graft for repair

6. The MOST common organism in acute otitis media of

older children and adults is:

a) Staphylococcus

b) Streptococcus

c) Hemophilus influenzae

d) Klebsiella pneumoniae

e) Pseudomonas

7. A 65-year-old white male who has been smoking pipes

since early adulthood notes a small patch of white on the

lateral anterior portion of the tongue. The patch is not

painful for the first month, but gradually becomes more

painful as it begins to enlarge and ulcerate. The MOST

likely diagnosis is?

a) benign nonspecific ulceration

b) leukoplakia (benign)

c) epulis

d) carcinoma of the tongue

e) ranula of the tongue

8. The diagnosis in question 7 may be confirmed by:

a) a positive Wasserman test

b) a positive lupus erythematosus (LE) preparation

c) a biopsy of the lesion

d) diagnostic mandibular and maxillary x-rays

e) observation of further progression of the disease

9. A cholesteatoma is:

a) an atherosclerotic lesion

b) a dermal collection of cholesterol salts

c) epithelial debris in the middle ear

d) a yellow papule beneath the oral tongue

e) retained cerumen

10. Small, malignant tumours of the larynx that are intrinsic in

origin and have not spread beyond the larynx are BEST

treated by:

a) irradiation

b) laryngofissure

c) total laryngectomy

d) total laryngectomy and radical neck dissection

e) radium needle implants

11. Clinical features of facial fractures frequently include all of

the following EXCEPT:

a) deformity

b) facial nerve paralysis

c) anesthesia over areas of trigeminal branch

distribution

d) ocular disparity

e) malocclusion of the teeth

12. The MOST sensitive test for nasal fracture is:

a) history

b) physical diagnosis

c) plain x-ray studies

d) magnetic resonance imaging

e) computed tomography (CT) scanning

13. Mixed tumours of the salivary gland:

a) are most common in the submaxillary gland

b) are usually malignant

c) are most common in the parotid gland

d) usually cause facial paralysis

e) are associated with calculi

14. In epistaxis, what percentage of the cases will respond to

ten minutes of direct pressure?

a) 10%

b) 30%

c) 70%

d) 90%

e) 0%

15. Which of the following is NOT a cause for conductive

hearing loss?

a) otitis media

b) otosclerosis

c) noise-induced hearing loss

d) perforation of the tympanic membrane

e) ossicular chain disruption

32 – Sample Questions MCCQE 2002 Review Notes

OTOLARYNGOLOGY . . . CONT.

16. Conductive hearing losses are usually reversible. Which

of the following conditions is reversible by surgical

treatment?

a) otosclerosis

b) presbycusis

c) sudden hearing loss

d) ototoxicity

e) meningitis

17. What is the BEST treatment for most cases of

sensorineural hearing loss associated with

aging (presbycusis)?

a) nothing

b) hearing aid

c) ear trumpet

d) diuretic therapy

e) labyrinthectomy

18. The MOST common benign lesion of the external

ear is:

a) melanoma

b) chondrodermatitis nodularis chronicus helicus

c) cerumenoma

d) actinic keratosis

e) exostosis of the canal

19. MOST of the infectious and/or inflammatory diseases

involving the middle ear space are secondary to:

a) ciliary dyskinesia

b) resistant pathogens

c) eutstachian tube dysfunction

d) tobacco abuse

e) allergic diathesis

20. Acute otitis is:

a) a rare condition

b) the most common reason ill children visit the doctor

c) usually not accompanied by pain and fever

d) caused by coliform bacteria

e) treated by placing ventilating tubes

21. All of the following ar indications for tonsillectomy

EXCEPT:

a) six to seven episodes of tonsillitis in 1 year

b) airway obstruction secondary to tonsillar

hypertrophy

c) repeat ear and sinus infections

d) recurrent peritonsillar abscess

e) very large asymmetric tonsil in an adult

22. The MOST common cause for infant stridor, accounting

or 60% of the cases, is:

a) subglottic hemangioma

b) vocal cord paralysis

c) laryngomalacia

d) congenital webs

e) laryngeal cleft

23. What is the MOST common cause of acquired subglottic

stenosis?

a) motor vehicle trauma

b) prolonged endotracheal intubation

c) chronic bronchitis

d) tracheoesophageal fistula

e) previous tracheal surgery

24. A 5-year-old child has persistent serous effusions in both

ears for 6 months after a routine acute infection. He has a

40-dB condutive heraring loss in both ears and has been

having trouble in school. What would be the BEST

treatment for this child?

a) observe the child for another 3 months

b) prescribe amoxicillin for 10 days

c) recommend hearing aids

d) place ventilating tubes

e) prescribe prophylactic antibiotics for 3 months

25. A 3-year-old child has has eight episodes of acute otitis

media in 6 months and has difficulty resolving the

effusions between infections. What should be done to

effectively eliminate the infections?

a) continuing treating each infection as it arises

b) place ventilating tubes

c) prescribe prophylactic antibiotics for 6 months

d) remove the tonsils

e) give IV antibiotics for 4 weeks after infectious

disease consultation

26. The following clinical entities are common causes for

tinnitus EXCEPT:

a) high-frequency hearing loss

b) Ménière’s disease

c) ototoxic drugs

d) loud noise exposure

e) acute otitis media

27. Vertigo is very common in all of the following conditions

EXCEPT:

a) vestibular neuritis

b) Ménière’s disease

c) presbycusis

d) viral labyrinthitis

e) benign paroxysmal positional vertigo

28. The fastest, safest means of establishing a surgical

airway is:

a) endoscopic intubation

b) tracheotomy under local anesthesia

c) tracheotomy under general anesthesia

d) cricothyrotomy

e) puncture through the thyroid membrane

MCCQE 2002 Review Notes Sample Questions – 33

PEDIATRICS

1. A 6 month-old infant presents in the winter with fever, cough,

wheezing, tachypnea and decreased appetite. A chest

radiograph shows hyperaeration and streaky perihilar infiltrates

bilaterally. You diagnose bronchopneumonia. Which organism

would most likely be causing this child's infection?

a) Chlamydia pneumoniae

b) Mycoplasma pneumoniae

c) Streptococcus pneumoniae

d) Haemophilus influenzae

e) respiratory syncytial virus

The following case pertains to questions 2 and 3:

A 12 month-old girl is brought to the emergency department for the

second time in 2 days for vomiting and passage of 8 to 10 watery stools

per day.

2. Of the following, which provides the best estimate of the

patient’s volume deficit:

a) weight change since the beginning of the illness

b) hydration of mucous membranes, skin turgor, and

level of consciousness

c) pulse, blood pressure, and peripheral capillary filling time

d) serum electrolytes

e) serum urea nitrogen and creatinine levels

3. The patient has lost 0.6 kg. She is moderately lethargic and has

dry mucous membranes and reduced skin turgor. Blood pressure is

80/40 mm Hg, and pulse is 120 per minute; capillary refill is

reasonably brisk. Lab studies reveal: sodium 131, potassium 4.8,

chloride 101, bicarbonate 16 mEq/L, urea nitrogen 24 mg/dL and

creatinine 0.6 mg/dL. The best strategy for managing this child

is to:

a) hospitalize for administration of IV fluid therapy

b) administer an oral rehydrating solution while the

child is under medical supervision for 4-6 hours

c) instruct the parents about oral rehydration at home

d) hospitalize after giving 20 mL/kg of 0.9% saline IV

e) instruct parents on use of soy formula

4. Which of the following cases is most suggestive of child abuse?

a) a child who clings to her parent but shies away from the

emergency physician.

b) a parent who refuses to leave his/her child alone with the

physician, despite the physician’s repeated requests.

c) a parent who claims that his child broke her arm after falling off

her bike.

d) a child with recurrent urinary tract infections despite antibiotic

prophylaxis.

e) a child with old-looking bruises on both elbows and shins.

5) A 14 year-old girl has refused to go to school 3 times in the past

4 months. She says, "My tummy hurts," but she cannot point to

where it bothers her. Her appetite is good, her bowel movements

are normal, and she is sleeping well at night. She is a

healthy-looking girl with no abnormal findings on physical exam.

What is the most likely diagnosis?

a) benign abdominal mass compressing her duodenum

b) diaphragmatic hernia

c) functional abdominal pain

d) gastroesophageal reflux disease

e) early peptic ulcer disease

6. Which of the following investigations is most helpful in the

assessment of a child presenting with an acute asthma attack, who

responds poorly to treatment?

a) white cell count and differential

b) arterial blood gases

c) chest x-ray

d) pulmonary function tests

e) sweat chloride test

7. A 6 year-old boy is brought to emergency with a 5 day history of

fever, cough, and poor appetite, but no vomiting. On exam, he

appears unwell, is febrile, and has crusty nasal discharge and a

wet-sounding cough. Chest x-ray reveals a pulmonary infiltrate in the

right middle lobe. He is admitted to hospital for pneumonia. What is

the best management for this child?

a) observe for 24 hours with IV fluids only

b) observe for 24 hours with IV fluids and acetaminophen

c) give oral amoxicillin and acetaminophen

d) give IV ampicillin and oral acetaminophen

e) give oral erythromycin and acetaminophen

8. In comparing breast milk and formula, which of the following

statements is NOT correct:

a) breast milk has a higher percentage of protein

b) breast milk has whey:casein ratio of 60:40

c) breast milk contains leukocytes, complement and lysozymes

d) breast milk has a lower concentration of iron than iron

fortified formulae

e) breast milk has an optimal calcium:phosphorus ratio of 2:1

9. Most umbilical hernias in children:

a) need strapping

b) resolve spontaneously

c) require elective surgery

d) require immediate surgery

e) are associated with a higher incidence of inguinal hernias

10. A newborn male spits up his first feeding and develops bilious

emesis with subsequent feedings. On physical exam he appears ill,

has a scaphoid abdomen and absent bowel sounds. Abdominal x-ray

shows air in the proximal small bowel, but a paucity of air in the

distal digestive tract. The most likely cause for this infant's vomiting

and clinical finding is:

a) antral web

b) choledochal cyst

c) Hirschsprung disease

d) tracheoesophageal fistula

e) volvulus

11. Breastfeeding is contraindicated if the mother has:

a) acute EBV infection

b) acute HepA infection

c) chronic HepB infection

d) asymptomatic HIV infection

e) none of the above

PEDIATRICS . . . CONT.

12. A 10 month-old child develops low-grade fever and sunburn-like

erythema over much of the body, but most prominently

in the intertriginous areas. Within 36 hours, sheet-like

desquamation is noted in the flexures and around the mouth.

The mucous membranes are spared. Which of the following is

the most likely diagnosis:

a) Kawasaki disease

b) staphylococcal scalded skin syndrome

c) Stevens-Johnson syndrome

d) toxic epidermal necrolysis

e) toxic shock syndrome

13. Which of the following is a contraindication to breast feeding

in Canada:

a) allergic disease in the family

b) infantile diarrhea

c) marijuana smoking

d) HIV infection

e) maternal use of ibuprofen

14. Central cyanosis in the newborn infant is most often caused by:

a) congenital heart disease

b) lung disease

c) central nervous system disease

d) methemoglobinemia

e) hypoglycemia

15. In a premature infant who is suspected of having necrotizing

enterocolitis (NEC), each of the following is correct EXCEPT:

a) septicemia is associated with an increased risk of NEC

b) NEC is thought to be caused by systemic hypertension

c) the finding of air in the portal vein indicates severe illness

d) respiratory distress increases the risk of NEC

e) Apgar scores inversely correlate with the risk of NEC

16. Which of the following is the most significant risk factor for the

development of childhood asthma:

a) family social background

b) parental asthma

c) stress in the family

d) parental smoking

e) presence of pets in the house

17. Which of the following statements about sickle cell disease is true?

a) all patients with sickle cell disease have a homozygous HbSS

genotype

b) sickle cell disease causes a severe chronic anemia that is not

routinely transfusion dependent

c) patients have increased susceptibility to infection by

nonencapsulated organisms

d) patients usually present with sickle cell crises within one

month of age

e) splenic dysfunction usually does not occur until the child

enters his/her teens

18. Which of the is more characteristic of platelet abnormalities and

not coagulation defects?

a) hematomas

b) hemarthrosis

c) petechiae

d) minimal bleeding from small cuts

e) positive family history

19. Which of the following statements is true?

a) cancer is the second most common cause of death in children

b) Hodgkin's lymphoma is the most common childhood cancer

c) Hyperploidy in leukemic cells is a poor prognostic indicator

d) Wilm's tumor is rarely associated with other congenital

abnormalities

e) neuroblastomas usually occur in late adolescence

20. A 5 year-old girl with hypogammaglobulinemia and absent

immunoglobulin A (IgA) receives infusions of immune

globulin monthly. Shortly after her most recent infusion

began, she developed hypotension, wheezing, and several

urticarial lesions. Which of the following best explains her

reaction?

a) anaphylactic reaction due to IgE anti-IgA antibodies

b) gram-negative sepsis due to contaminated immune

globulin

c) idiosyncratic reaction due to rapid infusion of immune

globulin

d) serum sickness reaction from foreign serum in the immune

globulin

e) transfusion reaction due to ABO incompatibility

21. When prophylactic antibiotic therapy is used for tooth

extraction in a patient with a ventricular septal defect:

a) therapy is started 24 hours prior to the procedure

b) a throat swab should be taken prior to the procedure

c) a second generation cephalosporin (cefuroxime) is the

therapy of choice

d) therapy is given for one week following the extraction

e) erythromycin is the drug of choice for those allergic to penicillin

22. Which of the following statements about stuttering in a

4 year-old is incorrect:

a) it is characterized by intermittent difficulty in producing a

smooth flow of speech

b) it is more than 3 times more frequent in girls than in boys

c) it is exacerbated by anxiety

d) more than 30% of children who stutter recover spontaneously

e) none of the above

23. A woman who is positive for hepatitis B surface antigen

(HBsAg), but negative for hepatitis B antigen (HBeAg),

delivers at term. What would be the best management for

this woman's infant?

a) administer gamma globulin intramuscularly immediately

and at 1 month of age

b) administer hepatitis B (HB) vaccine immediately and at 1

month and 6 months of age

c) administer hepatitis B immune globulin (HBIG) if cord

blood is positive for HBsAg

d) administer HBIG and HB vaccine immediately, and HB

vaccine again at 1 month and 6 months of age

e) advise mother that breastfeeding is contraindicated

24. Regarding sexual abuse of children, each of the following

statements is true EXCEPT:

a) no genital injury is found in the majority of patients

b) father-daughter incest is more common than brother-sister

incest

c) most assailants are unknown to the victim

d) laboratory findings usually show no presence of sperm

e) half of the abused children come from single parent families

34 – Sample Questions MCCQE 2002 Review Notes

MCCQE 2002 Review Notes Sample Questions – 35

PEDIATRICS . . . CONT.

25. The viral infection most likely to cause CNS involvement and focal

neurological findings is :

a) coxsackievirus

b) herpes simplex

c) enterovirus

d) rabies

e) rhinovirus

26. Which of the following predisposes infants to chronic otitis media?

a) bottle-feeding in upright position

b) abnormal tympanic membrane formation

c) environmental factors such as daycare and passive smoking

d) allergies

e) none of the above

27. All infants less than 3 months of age who have fever and no

localizing signs should have all of the following evaluations

EXCEPT:

a) blood cultures

b) careful history and physical exam

c) chest x-ray

d) CBC (with differential)

e) urine culture

28. All the following statements regarding transient tachypnea of the

newborn (TTN) is true EXCEPT:

a) infants born by C-section are at increased risk for

developing TTN

b) residual pulmonary function disability is common among

infants who have TTN

c) the incidence of TTN is higher than Respiratory Distress

Syndrome (RDS) among term infants

d) TTN shows marked improvement with 12-24 hrs

29. Antibiotic prophylaxis against infective endocarditis is required for

all of the following EXCEPT:

a) rheumatic valve lesions

b) prosthetic heart valves

c) isolated secundum atrial septal defect

d) pacemaker leads

e) patent ductus arteriosus

30. Causes of microcytic anemia include all of the following EXCEPT:

a) excessive cow's milk intake

b) iron deficiency

c) folic acid deficiency

d) thalassemias

e) lead poisoning

31. The latest time after the onset of group A ß hemolytic

Streptococcus pharyngitis that initiation of penicillin therapy can be

expected to prevent acute rheumatic fever is:

a) 24 hrs

b) 48 hrs

c) 96 hrs

d) 9 days

e) 2 weeks

32. A 16 year-old male has a 4 day illness of abrupt onset consisting of

fever, sore throat, rhinorrhea, cough and mild abdominal pain. He

has a red tonsillopharyngeal area, a whitish exudate on the tonsils

and tender anterior cervical nodes. The most likely cause of his

infection is:

a) adenovirus

b) Group A beta-hemolytic Streptococcus

c) Epstein-Barr Virus

d) Neisseria gonorrhea

e) Staphylococcus

33. A 13 month-old infant boy has chronic diarrhea, poor appetite,

irritability and growth failure. He had been well previously,

developing until diarrhea began at 3 months of age. Findings

include weight loss less than 5th percentile and length at 25th

percentile, cachectic appearance, wasted extremities and

protuberant abdomen. His labs are: albumin 23, protein 40. Stool

positive for reducing sugars and negative for enteric pathogens

and ova and parasites. These findings are most consistent with

a) Celiac disease

b) cow milk allergy

c) Crohn’s disease

d) cystic fibrosis

e) none of the above

34. Which of the following is most likely to provide the basis for

making the correct diagnosis in an infant or child who has failure to

thrive (FTT)?

a) blood chemistries

b) cultures

c) history and physical

d) radiograph studies

e) none of the above

35. A 8 year-old boy has had paroxysmal abdominal pain since his

parents separated 6 months ago. Which of the following symptoms

would support an organic basis for his disease?

a) headaches accompanies the pain

b) pain located in the periumbilical region

c) pain awakens child at night

d) symptoms last < 1hr

e) none of the above

36. The most common cause of chronic diarrhea in a

6 month - 36 month old child is:

a) chronic non-specific diarrhea

b) disaccharidase deficiency

c) enteric infection

d) malabsorption

e) protein intolerance

37. Which of the following is NOT required for a diagnosis of juvenile

rheumatoid arthritis?

a) arthritis in at least one joint

b) arthritis lasting for at least 6 weeks

c) positive rheumatoid factor

d) onset before the age of 16

e) other causes of arthritis excluded

PEDIATRICS . . . CONT.

38. A child presents with bilateral shin pain. Which of the following

suggests that this is NOT growing pains?

a) pain is poorly localized

b) pain awakens the child at night

c) no fever or rash

d) pain abates with reassurance and massage

e) child may limp in the morning from stiffness

39. Which of the following steps is NOT indicated in the management

of croup?

a) keeping the child calm

b) hydration

c) antipyretics

d) antibiotics

e) humidified oxygen

40. Which of the following pairs shows the CORRECT stage of normal

development?

a) 6 months - pulls to stand

b) 12 months - pincer grasp

c) 18 months - handedness

d) 2 years - draws a cross

e) 3 years - tells a story

41. Which of the following is NOT characteristic of a functional

murmur?

a) pansystolic murmur

b) murmur varies with position

c) variably split S2

d) murmur becomes louder with fever

e) no extra clicks

42. Which of the following is the most common congenital heart lesion

in children?

a) atrial septal defect

b) ventricular septal defect

c) patent ductus arteriosus

d) Tetralogy of Fallot

e) coarctation of the aorta

43. Choose the INCORRECT statement about breastfeeding:

a) for healthy term babies, breastfeeding is recommended over

formula feeding

b) breastmilk contains more amino acids than cow’s milk.

c) exclusively breastfed babies should be supplemented with

iron after 6 months of age

d) assessment of adequate intake can be done by counting

the number of wet diapers in a day

e) an HIV-positive mother should not breastfeed

44. All of the following are characteristics of a child with Pervasive

Developmental Disorder (PDD) EXCEPT:

a) abnormal speech pattern

b) stereotypic behaviours, such as head-banging or hand-flapping

c) abnormal cognitive function

d) consuming interest in one topic or activity

e) tendency to reciprocate in peer interactions

45. All of the following are appropriate in the initial assessment of a

child with failure to thrive EXCEPT:

a) diet history

b) social history

c) measurement of height, weight and head circumference

d) growth hormone levels

e) bone age x-ray

46. Which of the following statements about a child with short stature

is CORRECT?

a) the bone age is delayed in a child with constitutional growth

delay

b) height crosses a major percentile line on the child’s growth

chart

c) weight is more affected than height in a child with an

endocrine deficiency

d) karyotyping part of the routine investigation of all children with

short stature

e) growth hormone replacement is helpful in a child with familial

growth delay

47. Which of the following developmental milestones in not a cause for

concern?

a) 15 month old not walking

b) 12 month old not talking

c) 12 month old who does not search for hidden objects

d) 3 year old unable to stand on one foot momentarily

e) 6 month old with a persistent grasp reflex

48. All of the following immunizations should be administered to a

7 year-old child who is a recent immigrant, with unknown

vaccination status EXCEPT:

a) tetanus

b) MMR

c) diptheria

d) pertussis

e) polio

49. A 3 year old girl has a 2 month history of left knee swelling and

morning stiffness. There is no history of fever or rash. The child

appears very healthy. The most likely tentative diagnosis is:

a) pauciarticular juvenile rheumatoid arthritis (JRA)

b) systemic lupus erythematosus (SLE)

c) rheumatic fever

d) Kawasaki’s disease

e) Henoch-Schonlein purpura

50. An 8 year old boy has had 2 episodes of loss of awareness. These

last 5 seconds and consist of eyes and head turning to one side.

EEG reveals 3Hz spike and wave activity. First line therapy would

include:

a) ACTH

b) phenytoin

c) carbamazepine

d) ethosuximide

e) phenobarbital

36

3 comments:

  1. MCCEE QBank is one among the most effective MCC examination preparation platform wherever thousands of users have found a fast and simple thanks to get sensible grades in MCC analysis examination.

    ReplyDelete
  2. Dr itua herbal medicine is mighty and genuine,I use to use Melissa oil and Lavender Healing herbs but result were just still the same.But using Doctor Itua herbal medicine cured my herpes & Hepatitis C final and I have being living for 3 years now No Sores.My herpes is truly gone.Contact him if you interested of his herbal medicine too Dr itua can as well cure the following....HIV...HERPES,,,,HEPATITIS,CANCER,Neuro-infectious Disease,Neuromuscular Diseases,Diabetes,Depression,Anxiety,Hemorrhoid,Yeast infection,Lupus,Shingles,Psoriasis,Back Pain,ALS, Get ex back, Tinnitus.drituaherbalcenter@gmail.com Or Website www.drituaherbalcenter.com

    ReplyDelete